2
0
Fork 0
mirror of https://github.com/MartinThoma/LaTeX-examples.git synced 2025-04-28 23:37:57 +02:00

Remove trailing spaces

The commands

find . -type f -name '*.md' -exec sed --in-place 's/[[:space:]]\+$//' {} \+

and

find . -type f -name '*.tex' -exec sed --in-place 's/[[:space:]]\+$//' {} \+

were used to do so.
This commit is contained in:
Martin Thoma 2015-10-14 14:25:34 +02:00
parent c578b25d2f
commit 7740f0147f
538 changed files with 3496 additions and 3496 deletions

View file

@ -18,7 +18,7 @@ Folgende Definition wurde dem Skript von Herrn Prof.~Dr.~Leuzinger für
Lineare Algebra entnommen:
\begin{definition}\xindex{Abbildung!affine}%
Es seien $V$ und $W$ $\mdk$-Vektorräume und $\mda(V)$ und $\mda(W)$ die
Es seien $V$ und $W$ $\mdk$-Vektorräume und $\mda(V)$ und $\mda(W)$ die
zugehörigen affinen Räume. Eine Abbildung $f:V \rightarrow W$ heißt \textbf{affin},
falls für alle $a, b \in V$ und alle $\lambda, \mu \in \mdk$ mit $\lambda + \mu = 1$ gilt:
\[f(\lambda a + \mu b) = \lambda f(a) + \mu f(b)\]
@ -36,13 +36,13 @@ Lineare Algebra entnommen:
\end{definition}
\begin{satz*}[Zwischenwertsatz]\xindex{Zwischenwertsatz}%
Sei $a<b$ und $f \in\ C[a, b]:=C([a, b])$, weiter sei $y_0 \in \mdr$ und
$f(a) < y_0 < f(b)$ oder $f(b) < y_0 < f(a)$. Dann existiert ein
Sei $a<b$ und $f \in\ C[a, b]:=C([a, b])$, weiter sei $y_0 \in \mdr$ und
$f(a) < y_0 < f(b)$ oder $f(b) < y_0 < f(a)$. Dann existiert ein
$x_0 \in [a, b]$ mit $f(x_0) = y_0$.
\end{satz*}
\begin{definition}\xindex{Eigenwert}\xindex{Eigenvektor}%
Sei $V$ ein Vektorraum über einem Körper $\mdk$ und $f: V \rightarrow V$ eine
Sei $V$ ein Vektorraum über einem Körper $\mdk$ und $f: V \rightarrow V$ eine
lineare Abbildung.
$v \in V \setminus \Set{0}$ heißt \textbf{Eigenvektor} $:\Leftrightarrow \exists \lambda \in \mdk: f(v) = \lambda v$.

View file

@ -39,10 +39,10 @@
\usepackage[left=10mm,right=10mm, top=2mm, bottom=10mm]{geometry}
\usepackage{../shortcuts}
\hypersetup{
pdfauthor = {Martin Thoma},
pdfkeywords = {Geometrie und Topologie},
pdftitle = {Fragen zu Definitionen}
\hypersetup{
pdfauthor = {Martin Thoma},
pdfkeywords = {Geometrie und Topologie},
pdftitle = {Fragen zu Definitionen}
}
\allowdisplaybreaks
@ -73,7 +73,7 @@ $f:|K| \rightarrow |L|$
mit $f(\Delta) \notin L$?}
\section*{18.) ÜB 1, Aufgabe 2}
\underline{Vor.:} Es sei $(X, d)$ ein metrischer Raum, $A \subseteq X$.
\underline{Vor.:} Es sei $(X, d)$ ein metrischer Raum, $A \subseteq X$.
Weiter bezeichne $\fT$ die von $d$ auf $X$ erzeugte Topologie $\fT'$, die von
der auf $A \times A$ eingeschränkten Metrik $d|_{A \times A}$ erzeugte Topologie.
@ -87,7 +87,7 @@ Sei $U \in \fT|_A = \Set{V \cap A | V \in \fT}$.\\
Dann ex. also $V \in \fT$ mit
$U = V \cap A$.\\
Sei $x \in U$.\\
Da $V \in \fT$, ex. nach Bemerkung~3 ein $r > 0$ mit
Da $V \in \fT$, ex. nach Bemerkung~3 ein $r > 0$ mit
\begin{align*}
\fB_r(x) := \Set{y \in X | d(x,y) < r} &\subseteq V\\
@ -123,7 +123,7 @@ Da $x \in U$ beliebig gewählt war gilt: $\fT|_A \subseteq \fT'$
\[m_g: X \rightarrow X, x \mapsto g \circ x\]
ein Homöomorphismus ist.
\item Ist $G$ eine topologische Gruppe, so heißt die Gruppenoperation $\circ$
\textbf{stetig}\xindex{Gruppenoperation!stetige}, wenn
\textbf{stetig}\xindex{Gruppenoperation!stetige}, wenn
$\circ: G \times X \rightarrow X$ stetig ist.
\end{defenum}
\end{definition}
@ -172,7 +172,7 @@ $\Rightarrow$ Widerspruch
Da $r > 0$ ist $H_1$ nicht leer, da $r \in \mdr$ ist $H_2$ nicht leer.
\underline{Zu zeigen:} $\forall A \in H_i$, $B \in H_j$ mit
$i,j \in \Set{1,2}$ gilt:
$i,j \in \Set{1,2}$ gilt:
$\overline{AB} \cap g \neq \emptyset \Leftrightarrow i \neq j$\\
\enquote{$\Leftarrow$}: Da $d_\mdh$ stetig ist, folgt diese Richtung
direkt. Alle Punkte in $H_1$ haben einen Abstand von $m$ der kleiner
@ -189,7 +189,7 @@ $\Rightarrow$ Widerspruch
\[\mdh = \underbrace{\Set{z \in \mdh | \Re(z) < x}}_{=: H_1 \text{ (Links)}} \dcup \underbrace{\Set{z \in \mdh | \Re(z) > x}}_{=: H_2 \text{ (Rechts)}}\]
\underline{Zu zeigen:} $\forall A \in H_i$, $B \in H_j$ mit
$i,j \in \Set{1,2}$ gilt:
$i,j \in \Set{1,2}$ gilt:
$\overline{AB} \cap g \neq \emptyset \Leftrightarrow i \neq j$\\
\enquote{$\Leftarrow$}: Wie zuvor mit dem Zwischenwertsatz.
@ -207,7 +207,7 @@ $\Rightarrow$ Widerspruch
\begin{enumerate}
\item Deformationsretrakt: Das hatten wir nicht in der Vorlesung, oder? Ich meine mich zwar an das Wort zu erinnern (aus einem Übungsblatt? Einem Tutorium?) Könntest du bitte nochmals erklären was das ist?
Das ist zwar auf Blatt 7 und 8 vorgekommen, aber sonst nie.
\item Damit verbunden: Was genau ist eine "Einbettung"?
\item Damit verbunden: Was genau ist eine "Einbettung"?
\item Was bedeutet der Pfeil: $f:S^1 \hookrightarrow \mdr^2\;\;\;$ Einbettung der Kreislinie in die Ebene
\item Was ist eine Inklusionsabbildung?
\item Was ist ein Homotopietyp? (Ist das eventuell die Anzahl der Homotopieklassen?)

View file

@ -11,7 +11,7 @@
\item Ist $I$ eine Menge und $U_i \in \fT$ für jedes $i \in I$,
so ist $\displaystyle \bigcup_{i \in I} U_i \in \fT$
\end{defenumprops}
Die Elemente von $\fT$ heißen \textbf{offene Teilmengen} von $X$.
Die Elemente von $\fT$ heißen \textbf{offene Teilmengen} von $X$.
$A \subseteq X$ heißt \textbf{abgeschlossen}, wenn $X \setminus A$ offen ist.
\end{definition}
@ -44,7 +44,7 @@ Auch gibt es Mengen, die sowohl abgeschlossen als auch offen sind.
\item Jeder metrische Raum $(X, d)$ ist auch ein topologischer Raum.
\item Für eine Menge $X$ heißt $\fT_{\ts{Diskret}} = \powerset{X}$ \textbf{diskrete Topologie}\xindex{Topologie!diskrete}.
\item $X :=\mdr, \fT_Z := \Set{U \subseteq \mdr | \mdr \setminus U \text{ endlich}} \cup \Set{\emptyset}$ heißt \textbf{Zariski-Topologie} \xindex{Topologie!Zariski}\\
Beobachtungen:
Beobachtungen:
\begin{itemize}
\item $U \in \fT_Z \gdw \exists f \in \mdr[X]$, sodass $\mdr \setminus U = V(f) = \Set{x \in \mdr | f(x) = 0}$
\item Es gibt keine disjunkten offenen Mengen in $\fT_Z$.
@ -77,10 +77,10 @@ Auch gibt es Mengen, die sowohl abgeschlossen als auch offen sind.
\begin{beispiel}
\begin{bspenum}
\item Sei $X = \mdr$ mit euklidischer Topologie und
$M = \mdq$. Dann gilt: $\overline{M} = \mdr$ und
\item Sei $X = \mdr$ mit euklidischer Topologie und
$M = \mdq$. Dann gilt: $\overline{M} = \mdr$ und
$M^\circ = \emptyset$
\item Sei $X = \mdr$ und $M=(a,b)$. Dann gilt:
\item Sei $X = \mdr$ und $M=(a,b)$. Dann gilt:
$\overline{M} = [a,b]$
\item Sei $X = \mdr, \fT = \fT_Z$ und $M = (a,b)$. Dann gilt:
$\overline{M} = \mdr$
@ -102,14 +102,14 @@ Auch gibt es Mengen, die sowohl abgeschlossen als auch offen sind.
\begin{beispiel}[Basis und Subbasis]
\begin{bspenum}
\item Jede Basis ist auch eine Subbasis, z.B.\\
$S=\Set{ (a,b) | a,b \in \mdr, a<b }$ ist für $\mdr$ mit der
$S=\Set{ (a,b) | a,b \in \mdr, a<b }$ ist für $\mdr$ mit der
Standardtopologie sowohl Basis als auch Subbasis.
\item Gegeben sei $X = \mdr^n$ mit euklidischer Topologie $\fT$. Dann ist
\[\fB = \Set{B_r(x) | r \in \mdq_{> 0}, x \in \mdq^n}\]
ist eine abzählbare Basis von $\fT$.
\item Sei $(X, \fT)$ ein topologischer Raum mit
\item Sei $(X, \fT)$ ein topologischer Raum mit
$X = \Set{0,1,2}$ und $\fT = \Set{\emptyset, \Set{0}, \Set{0,1}, \Set{0,2}, X}$.\\
Dann ist $\calS = \Set{\emptyset, \Set{0,1}, \Set{0,2}}$ eine Subbasis von
Dann ist $\calS = \Set{\emptyset, \Set{0,1}, \Set{0,2}}$ eine Subbasis von
$\fT$, da gilt:
\begin{itemize}
\item $\calS \subseteq \fT$
@ -132,11 +132,11 @@ Auch gibt es Mengen, die sowohl abgeschlossen als auch offen sind.
Sei $(X, \fT)$ ein topologischer Raum und $Y \subseteq X$.\\
$\fT_Y := \Set{U \cap Y | U \in \fT}$ ist eine Topologie auf $Y$.
$\fT_Y$ heißt \textbf{Teilraumtopologie} und $(Y, \fT_Y)$ heißt ein
$\fT_Y$ heißt \textbf{Teilraumtopologie} und $(Y, \fT_Y)$ heißt ein
\textbf{Teilraum} von $(X, \fT)$.
\end{definition}
Die Teilraumtopologie wird auch \textit{Spurtopologie} oder
Die Teilraumtopologie wird auch \textit{Spurtopologie} oder
\textit{Unterraumtopologie} genannt.
%%%%%%%%%%%%%%%%%%%%%%%%%%%%%%%%%%%%%%%%%%%%%%%%%%%%%%%%%%%%%%%%%%%%%
@ -190,14 +190,14 @@ Die Teilraumtopologie wird auch \textit{Spurtopologie} oder
\begin{beispiel}
$X = \mdr, a \sim b :\Leftrightarrow a-b \in \mdz$
\input{figures/number-ray-circle-topology}
$0 \sim 1$, d.~h. $[0] = [1]$
\end{beispiel}
\begin{beispiel}\xindex{Torus}%
Sei $X = \mdr^2$ und $(x_1, y_1) \sim (x_2, y_2) \gdw x_1 - x_2 \in \mdz$
Sei $X = \mdr^2$ und $(x_1, y_1) \sim (x_2, y_2) \gdw x_1 - x_2 \in \mdz$
und $y_1 - y_2 \in \mdz$. Dann ist $X /_\sim$ ein Torus.
\end{beispiel}
@ -234,7 +234,7 @@ Die Teilraumtopologie wird auch \textit{Spurtopologie} oder
\begin{definition}\xindex{Isometrie}\label{def:Isometrie}%
Seien $(X, d_X)$ und $(Y, d_Y)$ metrische Räume und $\varphi: X \rightarrow Y$
eine Abbildung mit
eine Abbildung mit
\[\forall x_1, x_2 \in X: d_X(x_1, x_2) = d_Y(\varphi(x_1), \varphi(x_2)) \]
Dann heißt $\varphi$ eine \textbf{Isometrie} von $X$ nach $Y$.
@ -252,7 +252,7 @@ Die Teilraumtopologie wird auch \textit{Spurtopologie} oder
0 & \text{falls } x=y\\
1 & \text{falls } x \neq y
\end{cases}\]
die \textbf{diskrete Metrik}. Die Metrik $d$ induziert die
die \textbf{diskrete Metrik}. Die Metrik $d$ induziert die
\textbf{diskrete Topologie}.
\end{beispiel}
\clearpage
@ -280,7 +280,7 @@ Die Teilraumtopologie wird auch \textit{Spurtopologie} oder
\end{beispiel}
\clearpage
\begin{beispiel}[SNCF-Metrik\footnotemark]\xindex{Metrik!SNCF}
$X = \mdr^2$
$X = \mdr^2$
\input{figures/sncf-metrik}
\end{beispiel}
@ -293,7 +293,7 @@ Die Teilraumtopologie wird auch \textit{Spurtopologie} oder
\end{definition}
\begin{bemerkung}[Trennungseigenschaft]\label{Trennungseigenschaft}
Metrische Räume sind hausdorffsch, wegen
Metrische Räume sind hausdorffsch, wegen
\[d(x, y) > 0 \Rightarrow \exists \varepsilon > 0: \fB_\varepsilon(x) \cap \fB_\varepsilon(y) = \emptyset\]
\end{bemerkung}
@ -337,7 +337,7 @@ Die Teilraumtopologie wird auch \textit{Spurtopologie} oder
Sei $(x_n)$ eine konvergierende Folge und $x$ und $y$ Grenzwerte der Folge.
Da $X$ hausdorffsch ist, gibt es Umgebungen $U_x$ von $x$ und $U_y$
von $y$ mit $U_x \cap U_y = \emptyset$ falls $x \neq y$. Da
von $y$ mit $U_x \cap U_y = \emptyset$ falls $x \neq y$. Da
$(x_n)$ gegen $x$ und $y$ konvergiert, existiert ein
$n_0$ mit $x_n \in U_x \cap U_y$ für alle $n \geq n_0$
$\Rightarrow x = y \qed$
@ -345,14 +345,14 @@ Die Teilraumtopologie wird auch \textit{Spurtopologie} oder
\section{Stetigkeit}\index{Stetigkeit|(}
\begin{definition}
Seien $(X, \fT_X), (Y, \fT_Y)$ topologische Räume und
Seien $(X, \fT_X), (Y, \fT_Y)$ topologische Räume und
$f:X \rightarrow Y$ eine Abbildung.
\begin{defenum}
\item \label{def:stetigkeit} $f$ heißt \textbf{stetig}\xindex{Abbildung!stetige}
$:\gdw \forall U \in \fT_Y: f^{-1} (U) \in \fT_X$.
\item \label{def:homoeomorphismus} $f$ heißt \textbf{Homöomorphismus}\xindex{Homöomorphismus}, wenn $f$ stetig ist
und es eine
und es eine
stetige Abbildung $g: Y \rightarrow X$ gibt, sodass
$g \circ f = \id_X$ und $f \circ g = \id_Y$.
\end{defenum}
@ -378,9 +378,9 @@ Die Teilraumtopologie wird auch \textit{Spurtopologie} oder
\enquote{$\Rightarrow$}: Sei $x \in X, \varepsilon > 0$ gegeben
und $U := \fB_\varepsilon(f(x))$.\\
Dann ist $U$ offen in $Y$.\\
$\xRightarrow{\crefabbr{def:stetigkeit}} f^{-1}(U)$ ist
$\xRightarrow{\crefabbr{def:stetigkeit}} f^{-1}(U)$ ist
offen in $X$. Dann ist $x \in f^{-1}(U)$.\\
$\Rightarrow \exists \delta > 0$, sodass
$\Rightarrow \exists \delta > 0$, sodass
$\fB_\delta(x) \subseteq f^{-1} (U)$\\
$\Rightarrow f(\fB_\delta(x)) \subseteq U$\\
$\Rightarrow \Set{y \in X | d_X(x,y) < \delta} \Rightarrow$ Beh.
@ -415,18 +415,18 @@ Die Teilraumtopologie wird auch \textit{Spurtopologie} oder
\begin{figure}[htp]
\centering
\input{figures/topology-continuous-mapping}
\caption{Beispiel einer stetigen Funktion $f$, deren
\caption{Beispiel einer stetigen Funktion $f$, deren
Umkehrabbildung $g$ nicht stetig ist.}
\label{fig:nicht-stetige-umkehrabbildung}
\end{figure}
Die Umkehrabbildung $g$ ist nicht stetig, da $g^{-1}(U)$
nicht offen ist (vgl. \cref{fig:nicht-stetige-umkehrabbildung}).
\end{bspenum}
\end{beispiel}
\begin{bemerkung}[Verkettungen stetiger Abbildungen sind stetig]
Seien $X, Y, Z$ topologische Räume, $f:X \rightarrow Y$ und
Seien $X, Y, Z$ topologische Räume, $f:X \rightarrow Y$ und
$g:Y \rightarrow Z$ stetige Abbildungen.
Dann ist $g \circ f: X \rightarrow Z$ stetig.
@ -449,10 +449,10 @@ Die Teilraumtopologie wird auch \textit{Spurtopologie} oder
\begin{bemerkung}
\begin{bemenum}
\item \xindex{Homöomorphismengruppe}Für jeden topologischen Raum $X$ ist
\item \xindex{Homöomorphismengruppe}Für jeden topologischen Raum $X$ ist
\[\Homoo(X) := \Set{f: X \rightarrow X | f \text{ ist Homöomorphismus}}\]
eine Gruppe.
\item \xindex{Isometrie}Jede Isometrie $f:X \rightarrow Y$ zwischen metrischen
\item \xindex{Isometrie}Jede Isometrie $f:X \rightarrow Y$ zwischen metrischen
Räumen ist ein Homöomorphismus.
\item \xindex{Isometriegruppe}$\Iso(X) := \Set{f:X \rightarrow X | f \text{ ist Isometrie}}$ ist
eine Untergruppe von $\Homoo(X)$ für jeden
@ -462,7 +462,7 @@ Die Teilraumtopologie wird auch \textit{Spurtopologie} oder
\begin{bemerkung}[Projektionen sind stetig]
Seien $X, Y$ topologische Räume. $\pi_X: X \times Y \rightarrow X$
und $\pi_Y: X \times Y \rightarrow Y$ die Projektionen
und $\pi_Y: X \times Y \rightarrow Y$ die Projektionen
\[\pi_X: (x,y) \mapsto x \text{ und } \pi_Y: (x,y) \mapsto y\]
Wird $X \times Y$ mit der Produkttopologie versehen, so sind $\pi_X$
und $\pi_Y$ stetig.
@ -482,8 +482,8 @@ Die Teilraumtopologie wird auch \textit{Spurtopologie} oder
\end{bemerkung}
\begin{beweis}
Nach Definition ist
$U \subseteq \overline{X}$ offen $\gdw \pi^{-1}(U) \subseteq X$
Nach Definition ist
$U \subseteq \overline{X}$ offen $\gdw \pi^{-1}(U) \subseteq X$
offen. $\qed$
\end{beweis}
@ -498,7 +498,7 @@ sodass $\pi$ stetig wird.
S^n &= \Set{x \in \mdr^{n+1} | \|x\| = 1}\\
&= \Set{x \in \mdr^{n+1} | \sum_{i=1}^{n+1} x_i^2 = 1}
\end{align*}
\Obda sei $N = \begin{pmatrix}0\\ \vdots\\ 0\\1\end{pmatrix}$. Die
Gerade durch $N$ und $P$ schneidet die Ebene $H$ in genau einem
Punkt $\hat{P}$. $P$ wird auf $\hat{P}$ abgebildet.
@ -523,7 +523,7 @@ sodass $\pi$ stetig wird.
schneiden sich $L_P$ und $H$ in genau einem Punkt $\hat{P}$.
Es gilt: $f$ ist bijektiv und die Umkehrabbildung ist ebenfalls
stetig.
stetig.
\end{beispiel}
\index{Stetigkeit|)}
%%%%%%%%%%%%%%%%%%%%%%%%%%%%%%%%%%%%%%%%%%%%%%%%%%%%%%%%%%%%%%%%%%%%%
@ -533,7 +533,7 @@ sodass $\pi$ stetig wird.
\begin{definition}\xindex{Raum!zusammenhaengender@zusammenhängender}\xindex{Menge!zusammenhaengende@zusammenhängende}%
\begin{defenum}
\item Ein Raum $X$ heißt \textbf{zusammenhängend}, wenn es keine offenen,
nichtleeren Teilmengen $U_1, U_2$ von $X$ gibt mit
nichtleeren Teilmengen $U_1, U_2$ von $X$ gibt mit
$U_1 \cap U_2 = \emptyset$ und $U_1 \cup U_2 = X$.
\item Eine Teilmenge $Y \subseteq X$ heißt zusammenhängend, wenn $Y$
als topologischer Raum mit der Teilraumtopologie zusammenhängend ist.
@ -542,7 +542,7 @@ sodass $\pi$ stetig wird.
\begin{bemerkung}
$X$ ist zusammenhängend $\gdw$ Es gibt keine abgeschlossenen,
nichtleeren Teilmengen $A_1, A_2$ mit $A_1 \cap A_2 = \emptyset$
nichtleeren Teilmengen $A_1, A_2$ mit $A_1 \cap A_2 = \emptyset$
und $A_1 \cup A_2 = X$.
\end{bemerkung}
@ -553,20 +553,20 @@ sodass $\pi$ stetig wird.
\underline{Annahme}: $\mdr^n = U_1 \dcup U_2$ mit $\emptyset \neq U_1, U_2 \in \fT_{\ts{Euklid}}$ existieren.
Sei $x \in U_1, y \in U_2$ und $[x,y]$ die Strecke zwischen $x$
und $y$. Sei $V = [x,y]$. Nun betrachten wir $V \subsetneq \mdr^n$ als
und $y$. Sei $V = [x,y]$. Nun betrachten wir $V \subsetneq \mdr^n$ als
(metrischen) Teilraum mit der Teilraumtopologie $\fT_V$.
Somit gilt $U_1 \cap [x,y] \in \fT_V$ wegen der Definition der
Somit gilt $U_1 \cap [x,y] \in \fT_V$ wegen der Definition der
Teilraumtopologie.
Dann gibt es $z \in [x,y]$ mit $z \in \partial (U_1 \cap [x,y])$,
aber $z \notin U_1 \Rightarrow z \in U_2$. In jeder Umgebung von
aber $z \notin U_1 \Rightarrow z \in U_2$. In jeder Umgebung von
$z$ liegt ein Punkt von $U_1 \Rightarrow$ Widerspruch zu $U_2$ offen.
\item $\mdr \setminus \Set{0}$ ist nicht zusammenhängend, denn
$\mdr \setminus \Set{0} = \mdr_{< 0} \cup \mdr_{> 0}$
\item $\mdr^2 \setminus \Set{0}$ ist zusammenhängend.
\item $\mdq \subsetneq \mdr$ ist nicht zusammenhängend, da
\item $\mdq \subsetneq \mdr$ ist nicht zusammenhängend, da
$(\mdq \cap \mdr_{< \sqrt{2}}) \cup (\mdq \cap \mdr_{> \sqrt{2}}) = \mdq$
\item $\Set{x}$ ist zusammenhängend für jedes $x \in X$,
\item $\Set{x}$ ist zusammenhängend für jedes $x \in X$,
wobei $X$ ein topologischer Raum ist.
\item $\mdr$ mit Zariski-Topologie ist zusammenhängend.\xindex{Topologie!Zariski}
\end{bspenum}
@ -590,7 +590,7 @@ sodass $\pi$ stetig wird.
$\Rightarrow \overline{A} \subseteq A_2$\\
$\Rightarrow A_1 = \emptyset$\\
$\Rightarrow$ Widerspruch zu $A_1 \neq \emptyset$\\
$\Rightarrow A \cap A_1 \neq \emptyset$ und analog
$\Rightarrow A \cap A_1 \neq \emptyset$ und analog
$A \cap A_2 \neq \emptyset$\\
$\Rightarrow$ Widerspruch zu $A$ ist zusammenhängend. $ \qed$
\end{beweis}
@ -614,7 +614,7 @@ sodass $\pi$ stetig wird.
\begin{definition}\xindex{Zusammenhangskomponente}%
Sei $X$ ein topologischer Raum.
Für $x \in X$ sei $Z(x) \subseteq X$ definiert durch
\[Z(x) := \bigcup_{\mathclap{\substack{A \subseteq X \text{zhgd.}\\ x \in A}}} A\]
@ -647,7 +647,7 @@ sodass $\pi$ stetig wird.
\begin{align*}
\Rightarrow Z(x) \cup Z(y) &\subseteq Z(x) \Rightarrow Z(y) \subseteq Z(x)\\
&\subseteq Z(y) \Rightarrow Z(x) \subseteq Z(y)
\end{align*}
\end{align*}
\end{enumerate}
$\qed$
@ -678,7 +678,7 @@ sodass $\pi$ stetig wird.
Ein topologischer Raum $X$ heißt \textbf{kompakt}, wenn jede
offene Überdeckung von $X$
\[\fU = \Set{U_i}_{i \in I} \text{ mit } U_i \text{ offen in } X\]
eine endliche Teilüberdeckung
eine endliche Teilüberdeckung
\[\bigcup_{\mathclap{i \in J \subseteq I}} U_i = X \text{ mit } |J| \in \mdn\]
besitzt.
\end{definition}
@ -688,24 +688,24 @@ sodass $\pi$ stetig wird.
%%%%%%%%%%%%%%%%%%%%%%%%%%%%%%%%%%%%%%%%%%%%%%%%%%%%%%%%%%%%%%%%%%%%%
\begin{bemerkung}\label{abgeschlossen01IstKompakt}
Das Einheitsintervall $I := [0,1]$ ist kompakt bezüglich der
Das Einheitsintervall $I := [0,1]$ ist kompakt bezüglich der
euklidischen Topologie.
\end{bemerkung}
\begin{beweis}
Sei $(U_i)_{i \in J}$ eine offene Überdeckung von $I$.
Es genügt zu zeigen, dass es ein $\delta > 0$ gibt, sodass jedes
Es genügt zu zeigen, dass es ein $\delta > 0$ gibt, sodass jedes
Teilintervall der Länge $\delta$ von $I$ in einem der $U_i$ enthalten ist.
Wenn es ein solches $\delta$ gibt, kann man $I$ in endlich viele
Wenn es ein solches $\delta$ gibt, kann man $I$ in endlich viele
Intervalle der Länge $\delta$ unterteilen und alle $U_i$ in die endliche
Überdeckung aufnehmen, die Teilintervalle enthalten.
Angenommen, es gibt kein solches $\delta$. Dann gibt es für jedes
Angenommen, es gibt kein solches $\delta$. Dann gibt es für jedes
$n \in \mdn$ ein Intervall $I_n \subseteq [0,1]$ der Länge $\nicefrac{1}{n}$
sodass $I_n \subsetneq U_i$ für alle $i \in J$.
Sei $x_n$ der Mittelpunkt von $I_n$. Die Folge $(x_n)$ hat einen
Sei $x_n$ der Mittelpunkt von $I_n$. Die Folge $(x_n)$ hat einen
Häufungspunkt $x \in [0,1]$. Dann gibt es $i \in J$ mit $x \in U_i$.
Da $U_i$ offen ist, gibt es ein $\varepsilon > 0$, sodass $(x - \varepsilon, x + \varepsilon) \subseteq U_i$.
Dann gibt es $n_0$, sodass gilt:
@ -713,7 +713,7 @@ $\nicefrac{1}{n_0} < \nicefrac{\varepsilon}{2}$ und für unendlich viele\footnot
$n\geq n_0: |x - x_n| < \nicefrac{\varepsilon}{2}$, also $I_n \subseteq (x - \varepsilon, x + \varepsilon) \subseteq U_i$
für mindestens ein $n \in \mdn$.\footnote{Sogar für unendlich viele.}
$\Rightarrow$ Widerspruch
$\Rightarrow$ Widerspruch
Dann überdecke $[0,1]$ mit endlich vielen Intervallen $I_1, \dots, I_d$
der Länge $\delta$. Jedes $I_j$ ist in $U_{ij}$ enthalten.
@ -727,7 +727,7 @@ $\qed$
\item $\mdr$ ist nicht kompakt.
\item $(0,1)$ ist nicht kompakt.\\
$U_n = (\nicefrac{1}{n}, 1-\nicefrac{1}{n}) \Rightarrow \bigcup_{n \in \mdn} U_n = (0,1)$
\item $\mdr$ mit der Zariski-Topologie ist kompakt und jede
\item $\mdr$ mit der Zariski-Topologie ist kompakt und jede
Teilmenge von $\mdr$ ist es auch.\xindex{Topologie!Zariski}
\end{bspenum}
\end{beispiel}
@ -770,12 +770,12 @@ $\qed$
Die offenen Mengen $U_{x_0, y} \times V_{x_0, y}$ für festes $x_0$
und alle $y \in Y$ überdecken $\Set{x_0} \times y$. Da $Y$ kompakt
ist, ist auch $\Set{x_0} \times Y$ kompakt. Also gibt es
$y_1, \dots, y_{m(x_0)}$ mit
ist, ist auch $\Set{x_0} \times Y$ kompakt. Also gibt es
$y_1, \dots, y_{m(x_0)}$ mit
$\bigcup_{i=1}^{m(x_0)} U_{x_0, y_i} \times V_{x_0, y_i} \supseteq \Set{x_0} \times Y$.
Sei ${\color{blue} U_{x_0}} := \bigcap_{i=1}^{m(x)} U_{x_0, y_i}$.
Da $X$ kompakt ist, gibt es $x_1, \dots, x_n \in X$ mit
Da $X$ kompakt ist, gibt es $x_1, \dots, x_n \in X$ mit
$\bigcup_{j=1}^n U_{x_j} = X$\\
$\Rightarrow \bigcup_{j=1}^k \bigcup_{i=1}^{m(x_j)} \underbrace{\left ( U_{x_j, y_i} \times V_{x_j, y_i} \right)}_{\mathclap{\text{Ein grün-oranges Kästchen}}} \supseteq X \times Y$\\
$\Rightarrow \bigcup_j \bigcup_i W_i (x_j, y_i) = X \times Y \qed$
@ -789,7 +789,7 @@ $\qed$
\begin{beweis}
\underline{z.~Z.:} Komplement ist offen
Ist $X = K$, so ist $K$ abgeschlossen in $X$. Andernfalls sei
Ist $X = K$, so ist $K$ abgeschlossen in $X$. Andernfalls sei
$y \in X \setminus K$. Für jedes $x \in K$ seien $U_x$ bzw. $V_y$
Umgebungen von $x$ bzw. von $y$, sodass $U_x \cap V_y = \emptyset$.
@ -819,10 +819,10 @@ $\qed$
\begin{beweis}
Sei $(V_i)_{i \in I}$ offene Überdeckung von $f(K)$\\
$\xRightarrow{f \text{ stetig}} (f^{-1}(V_i))_{i \in I}$ ist offene Überdeckung von $K$\\
$\xRightarrow{\text{Kompakt}}$ es gibt $i_1, \dots, i_n$,
$\xRightarrow{\text{Kompakt}}$ es gibt $i_1, \dots, i_n$,
sodass $f^{-1}(V_{i_1}), \dots, f^{-1}(V_{i_n})$ Überdeckung von
$K$ ist.\\
$\Rightarrow f(f^{-1}( V_{i_1})), \dots, f(f^{-1}(V_{i_n}))$
$\Rightarrow f(f^{-1}( V_{i_1})), \dots, f(f^{-1}(V_{i_n}))$
überdecken $f(K)$.
Es gilt: $f(f^{-1}(V)) = V \cap f(X) \qed$
@ -839,7 +839,7 @@ $\qed$
Da $\mdr^n$ und $\mdc^n$ hausdorffsch sind, ist $K$ nach
\cref{hausdorffraumKompakteTeilmengeAbgeschlossen} abgeschlossen.
Nach Voraussetzung kann $K$ mit endlich vielen offenen Kugeln von
Nach Voraussetzung kann $K$ mit endlich vielen offenen Kugeln von
Radien 1 überdeckt werden $\Rightarrow K$ ist beschränkt.
\enquote{$\Leftarrow$} Sei $A \subseteq \mdr^n$ (oder $\mdc^n$)
@ -852,7 +852,7 @@ $\qed$
Nach \cref{kompaktTimesKompaktIstKompakt} und
\cref{abgeschlossen01IstKompakt} ist $W$ kompakt, also ist $A$
nach \cref{abgeschlossenInKomaktIstKompakt} auch kompakt.
Genauso ist $Z$ kompakt, weil
Genauso ist $Z$ kompakt, weil
\[\Set{z \in \mdc | |z| \leq 1}\]
homöomorph zu
\[\Set{(x,y) \in \mdr^2 | \|(x,y)\| \leq 1}\]
@ -864,11 +864,11 @@ $\qed$
%%%%%%%%%%%%%%%%%%%%%%%%%%%%%%%%%%%%%%%%%%%%%%%%%%%%%%%%%%%%%%%%%%%%%
\section{Wege und Knoten}\index{Knoten|(}
\begin{definition}\xindex{Weg}\xindex{Weg!geschlossener}\xindex{Weg!einfacher}%
Sei $X$ ein topologischer Raum.
Sei $X$ ein topologischer Raum.
\begin{defenum}
\item Ein \textbf{Weg} in $X$ ist eine stetige Abbildung $\gamma:[0,1] \rightarrow X$.
\item $\gamma$ heißt \textbf{geschlossen}, wenn $\gamma(1) = \gamma(0)$ gilt.
\item $\gamma$ heißt \textbf{einfach}, wenn $\gamma|_{[0,1)}$
\item $\gamma$ heißt \textbf{einfach}, wenn $\gamma|_{[0,1)}$
injektiv ist.
\end{defenum}
\end{definition}
@ -901,11 +901,11 @@ $\qed$
$A_1 \cup A_2 = X$. Sei $x \in A_1, y \in A_2, \gamma:[0,1] \rightarrow X$
ein Weg von $x$ nach $y$.
Dann ist $C:= \gamma([0,1]) \subseteq X$ zusammenhängend, weil
Dann ist $C:= \gamma([0,1]) \subseteq X$ zusammenhängend, weil
$\gamma$ stetig ist.
\[C = \underbrace{(C \cap A_1)}_{\ni x} \cup \underbrace{(C \cap A_2)}_{\ni y}\]
ist Zerlegung in nichtleere, disjunkte, abgeschlossene Teilmengen
$\Rightarrow$ Widerspruch
$\Rightarrow$ Widerspruch
\item Sei $X = \Set{(x,y) \in \mdr^2| x^2 + y^2 = 1 \lor y = 1 +2\cdot e^{-\frac{1}{10} x}}$.
@ -943,7 +943,7 @@ $\qed$
\end{beweis}
\begin{beispiel}[Hilbert-Kurve]\xindex{Hilbert-Kurve}%
Es gibt stetige, surjektive Abbildungen
Es gibt stetige, surjektive Abbildungen
$[0,1] \rightarrow [0,1] \times [0,1]$. Ein Beispiel ist die
in \cref{fig:hilbert-curve} dargestellte Hilbert-Kurve.
@ -952,7 +952,7 @@ $\qed$
\begin{definition}\xindex{Jordankurve}\xindex{Jordankurve!geschlossene}%
Sei $X$ ein topologischer Raum. Eine
\textbf{Jordankurve} in $X$ ist ein Homöomorphismus
\textbf{Jordankurve} in $X$ ist ein Homöomorphismus
$\gamma: [0,1] \rightarrow C \subseteq X$ bzw.
$\gamma: S^1 \rightarrow C \subseteq X$, wobei $C := \Bild{\gamma}$.
\end{definition}
@ -967,7 +967,7 @@ Jede Jordankurve ist also ein einfacher Weg.
\begin{figure}[htp]
\centering
\input{figures/topology-jordan}
\input{figures/topology-jordan}
\label{fig:jordan-kurvensatz}
\caption{Die unbeschränkte Zusammenhangskomponente wird häufig inneres, die beschränkte äußeres genannt.}
\end{figure}
@ -993,15 +993,15 @@ Jede Jordankurve ist also ein einfacher Weg.
\label{fig:knot-unknot}
}%
\subfloat[Kleeblattknoten]{
\includegraphics[width=0.2\linewidth, keepaspectratio]{figures/blue-trefoil-knot.png}
\includegraphics[width=0.2\linewidth, keepaspectratio]{figures/blue-trefoil-knot.png}
\label{fig:knot-trefoil}
}%
\subfloat[Achterknoten]{
\includegraphics[width=0.2\linewidth, keepaspectratio]{figures/blue-eight-knot.png}
\includegraphics[width=0.2\linewidth, keepaspectratio]{figures/blue-eight-knot.png}
\label{fig:knot-eight-knot}
}%
\subfloat[$6_2$-Knoten]{
\includegraphics[width=0.2\linewidth, keepaspectratio]{figures/blue-6-2-knot.png}
\includegraphics[width=0.2\linewidth, keepaspectratio]{figures/blue-6-2-knot.png}
\label{fig:knot-6-2}
}
@ -1014,20 +1014,20 @@ Jede Jordankurve ist also ein einfacher Weg.
Zwei Knoten $\gamma_1, \gamma_2: S^1 \rightarrow \mdr^3$ heißen
\textbf{äquivalent}, wenn es eine stetige Abbildung
\[H: S^1 \times [0,1] \rightarrow \mdr^3\]
gibt mit
gibt mit
\begin{align*}
H(z,0) &= \gamma_1(z) \;\;\;\forall z \in S^1\\
H(z,1) &= \gamma_2(z) \;\;\;\forall z \in S^1
\end{align*}
und für jedes
feste $t \in [0,1]$ ist
feste $t \in [0,1]$ ist
\[H_z: S^1 \rightarrow \mdr^3, z \mapsto H(z,t)\]
ein Knoten. Die Abbildung $H$ heißt \textbf{Isotopie} zwischen
$\gamma_1$ und $\gamma_2$.
\end{definition}
\begin{definition}\xindex{Knotendiagramm}%
Sei $\gamma: [0,1] \rightarrow \mdr^3$ ein Knoten, $E$ eine Ebene und
Sei $\gamma: [0,1] \rightarrow \mdr^3$ ein Knoten, $E$ eine Ebene und
$\pi: \mdr^3 \rightarrow E$ eine Projektion auf $E$.
$\pi$ heißt \textbf{Knotendiagramm} von $\gamma$, wenn gilt:
@ -1047,16 +1047,16 @@ Jede Jordankurve ist also ein einfacher Weg.
\begin{figure}[htp]
\centering
\subfloat[$\Omega_1$]{
\includegraphics[height=0.2\linewidth, keepaspectratio]{figures/reidemeister-move-1.png}
\includegraphics[height=0.2\linewidth, keepaspectratio]{figures/reidemeister-move-1.png}
\label{fig:reidemeister-1}
}\qquad\qquad%
\subfloat[$\Omega_2$]{
\includegraphics[height=0.2\linewidth, keepaspectratio]{figures/reidemeister-move-2.png}
\includegraphics[height=0.2\linewidth, keepaspectratio]{figures/reidemeister-move-2.png}
\label{fig:reidemeister-2}
}
\subfloat[$\Omega_3$]{
\includegraphics[height=0.2\linewidth, keepaspectratio]{figures/reidemeister-move-3.png}
\includegraphics[height=0.2\linewidth, keepaspectratio]{figures/reidemeister-move-3.png}
\label{fig:reidemeister-3}
}
@ -1069,15 +1069,15 @@ Jede Jordankurve ist also ein einfacher Weg.
\end{beweis}
\begin{definition}\xindex{Färbbarkeit}%
Ein Knotendiagramm heißt \textbf{3-färbbar},
wenn jeder Bogen von $D$ so mit einer Farbe gefärbt werden kann,
dass an jeder Kreuzung eine oder 3 Farben auftreten und alle 3
Ein Knotendiagramm heißt \textbf{3-färbbar},
wenn jeder Bogen von $D$ so mit einer Farbe gefärbt werden kann,
dass an jeder Kreuzung eine oder 3 Farben auftreten und alle 3
Farben auftreten.
\end{definition}
\begin{figure}[htp]
\centering
\includegraphics[height=0.3\linewidth, keepaspectratio]{figures/tricoloring.png}
\includegraphics[height=0.3\linewidth, keepaspectratio]{figures/tricoloring.png}
\caption{Ein 3-gefärber Kleeblattknoten}
\label{fig:treefoil-knot-three-colors}

View file

@ -15,7 +15,7 @@
Familie $(U_i, \varphi_i)_{i \in I}$ von Karten auf $X$,
sodass $\bigcup_{i \in I} U_i = X$.
\item $X$ heißt (topologische) $n$-dimensionale \textbf{Mannigfaltigkeit}\xindex{Mannigfaltigkeit},
wenn $X$ hausdorffsch ist, eine abzählbare Basis der
wenn $X$ hausdorffsch ist, eine abzählbare Basis der
Topologie hat und einen $n$-dimensionalen Atlas besitzt.
\end{defenum}
\end{definition}
@ -27,13 +27,13 @@ Anschaulich ist also ein $n$-dimensionale Mannigfaltigkeit lokal dem $\mdr^n$ ä
\end{bemerkung}
\begin{beweis}
Sei $(X, \fT)$ ein topologischer Raum und $(U, \varphi)$ mit $U \in \fT$
Sei $(X, \fT)$ ein topologischer Raum und $(U, \varphi)$ mit $U \in \fT$
und $\varphi:U \rightarrow V \subseteq \mdr^n$,
wobei $V$ offen und $\varphi$ ein Homöomorphismus ist, eine Karte auf $X$.
Da jede offene Teilmenge des $\mdr^n$ genauso mächtig ist wie der $\mdr^n$,
$\varphi$ als Homöomorphismus insbesondere bijektiv ist und Mengen, zwischen
denen eine Bijektion existiert, gleich mächtig sind, ist $U$ genauso mächtig
$\varphi$ als Homöomorphismus insbesondere bijektiv ist und Mengen, zwischen
denen eine Bijektion existiert, gleich mächtig sind, ist $U$ genauso mächtig
wie der $\mdr^n$. Da jede Mannigfaltigkeit mindestens eine Karte hat, muss
jede Mannigfaltigkeit $X$ mindestens so mächtig sein wie der $\mdr^n$. $\qed$
\end{beweis}
@ -59,8 +59,8 @@ Mannigfaltigkeiten können beliebig viele Elemente haben.
\begin{beispiel}[Mannigfaltigkeiten]
\begin{bspenum}
\item Jede offene Teilmenge $U \subseteq \mdr^n$ ist eine
$n$-dimensionale Mannigfaltigkeit mit einem Atlas aus
\item Jede offene Teilmenge $U \subseteq \mdr^n$ ist eine
$n$-dimensionale Mannigfaltigkeit mit einem Atlas aus
einer Karte.
\item $\mdc^n$ ist eine $2n$-dimensionale Mannigfaltigkeit
mit einem Atlas aus einer Karte:
@ -106,7 +106,7 @@ Mannigfaltigkeiten können beliebig viele Elemente haben.
Es gibt keine Umgebung von $0$ in $[0,1]$, die homöomorph
zu einem offenem Intervall ist.
\item $V_1 = \Set{(x,y) \in \mdr^2 | x \cdot y = 0}$ ist
keine Mannigfaltigkeit.
keine Mannigfaltigkeit.
Das Problem ist $(0,0)$. Wenn man diesen Punkt entfernt,
zerfällt der Raum in 4 Zusammenhangskomponenten.
@ -116,7 +116,7 @@ Mannigfaltigkeiten können beliebig viele Elemente haben.
Mannigfaltigkeit.
\item $X = (\mdr \setminus \Set{0}) \cup (0_1, 0_2)$ \label{bsp:mannigfaltigkeit8}
\[U \subseteq X \text{ offen } \gdw
\[U \subseteq X \text{ offen } \gdw
\begin{cases}
U \text{ offen in } \mdr \setminus \Set{0}, &\text{falls } 0_1 \notin U, 0_2 \in U\\
\exists \varepsilon > 0: (-\varepsilon, \varepsilon) \subseteq U &\text{falls } 0_1 \in U, 0_2 \in U
@ -128,7 +128,7 @@ Mannigfaltigkeiten können beliebig viele Elemente haben.
\underline{Aber:} $X$ ist nicht hausdorffsch!
Denn es gibt keine disjunkten Umgebungen von $0_1$ und
$0_2$.
\item \label{bsp:gln-ist-mf}\xindex{Gruppe!allgemeine lineare}$\GL_n(\mdr)$ ist eine Mannigfaltigkeit der Dimension
\item \label{bsp:gln-ist-mf}\xindex{Gruppe!allgemeine lineare}$\GL_n(\mdr)$ ist eine Mannigfaltigkeit der Dimension
$n^2$, weil offene Teilmengen von $\mdr^{n^2}$ eine
Mannigfaltigkeit bilden.
\end{bspenum}
@ -141,12 +141,12 @@ Mannigfaltigkeiten können beliebig viele Elemente haben.
Seien $X, Y$ $n$-dimensionale Mannigfaltigkeiten, $U \subseteq X$
und $V \subseteq Y$ offen, $\Phi: U \rightarrow V$ ein Homöomorphismus
$Z = (X \dcup Y) /_\sim$ mit der von $u \sim \Phi(u)\;\forall{u \in U}$
erzeugten Äquivalenzrelation und der von $\sim$ induzierten
erzeugten Äquivalenzrelation und der von $\sim$ induzierten
Quotiententopologie.
$Z$ heißt \textbf{Verklebung} von $X$ und $Y$ längs $U$ und $V$.
$Z$ besitzt einen Atlas aus $n$-dimensionalen Karten.
Falls $Z$ hausdorffsch ist, ist $Z$ eine $n$-dimensionale
Falls $Z$ hausdorffsch ist, ist $Z$ eine $n$-dimensionale
Mannigfaltigkeit.
\end{definition}
@ -210,7 +210,7 @@ Mannigfaltigkeiten können beliebig viele Elemente haben.
$G: U \rightarrow \mdr^n, \; u \mapsto (g(u), u)$
eine stetige Abbildung auf eine offene Umgebung $V$ von
$x$ in $X$ ist.
\end{enumerate}
\end{enumerate}
$\qed$
\end{beweis}
@ -244,8 +244,8 @@ Mannigfaltigkeiten können beliebig viele Elemente haben.
Sei $X$ ein Hausdorffraum mit abzählbarer Basis der Topologie.
$X$ heißt $n$-dimensionale \textbf{Mannigfaltigkeit mit Rand},
wenn es einen Atlas $(U_i, \varphi_i)$ gibt, wobei $U_i \subseteq X_i$
offen und $\varphi_i$ ein Homöomorphismus auf eine offene
Teilmenge von
offen und $\varphi_i$ ein Homöomorphismus auf eine offene
Teilmenge von
\[\mdr_{+,0}^n := \Set{(x_1, \dots, x_n) \in \mdr^n | x_n \geq 0}\]
ist.
\end{definition}
@ -275,7 +275,7 @@ $\mdr_{+,0}^n$ ist ein \enquote{Halbraum}\xindex{Halbraum}.
\begin{definition}\xindex{Rand}%
Sei $X$ eine $n$-dimensionale Mannigfaltigkeit mit Rand und
Atlas $\atlas$. Dann heißt
Atlas $\atlas$. Dann heißt
\[\partial X := \bigcup_{(U, \varphi) \in \atlas} \Set{x \in U | \varphi (x) = 0}\]
\textbf{Rand} von $X$.
\end{definition}
@ -322,7 +322,7 @@ Differenzierbare Mannigfaltigkeiten der Klasse $C^\infty$ werden auch
\textit{glatt} genannt.
\begin{definition}%
Sei $X$ eine differenzierbare Mannigfaltigkeit der Klasse $C^k$
Sei $X$ eine differenzierbare Mannigfaltigkeit der Klasse $C^k$
($k \in \mdn \cup \Set{\infty}$) mit Atlas $\atlas = (U_i, \varphi_i)_{i \in I}$.
\begin{defenum}
@ -330,10 +330,10 @@ Differenzierbare Mannigfaltigkeiten der Klasse $C^\infty$ werden auch
mit $\atlas$, wenn alle Kartenwechsel $\varphi \circ \varphi_i^{-1}$
und $\varphi_i \circ \varphi^{-1}$ ($i \in I$ mit $U_i \cap U \neq \emptyset$)
differenzierbar von Klasse $C^k$ sind.
\item Die Menge aller mit $\atlas$ verträglichen Karten auf
\item Die Menge aller mit $\atlas$ verträglichen Karten auf
$X$ bildet einen maximalen Atlas der Klasse $C^k$. Er
heißt \textbf{$C^k$-Struktur}\xindex{Ck-Struktur@$C^k$-Struktur} auf $X$.
Eine $C^\infty$-Struktur heißt auch \textbf{differenzierbare Struktur}\xindex{Struktur!differenzierbare}
auf $X$.
\end{defenum}
@ -357,7 +357,7 @@ Differenzierbare Mannigfaltigkeiten der Klasse $C^\infty$ werden auch
gibt, sodass $\psi \circ f \circ \varphi^{-1}$ stetig
differenzierbar von Klasse $C^k$ in $\varphi(x)$ ist.
\item $f$ heißt \textbf{differenzierbar}
(von Klasse $C^k$), wenn $f$ in jedem $x \in X$
(von Klasse $C^k$), wenn $f$ in jedem $x \in X$
differenzierbar ist.
\item $f$ heißt \textbf{Diffeomorphismus}\xindex{Diffeomorphismus},
wenn $f$ differenzierbar von Klasse $C^\infty$ ist und
@ -375,7 +375,7 @@ Differenzierbare Mannigfaltigkeiten der Klasse $C^\infty$ werden auch
\begin{beweis}
Seien $(U', \varphi')$ und $(V', \psi')$ Karten von $X$ bzw. $Y$
um $x$ bzw. $f(x)$ mit $f(U') \subseteq V'$.
$\Rightarrow \psi' \circ f \circ (\varphi')^{-1}$\\
$= \psi' \circ ( \psi^{-1} \circ \psi) \circ f \circ (\varphi^{-1} \circ \varphi ) \circ (\varphi')^{-1}$
@ -397,8 +397,8 @@ Differenzierbare Mannigfaltigkeiten der Klasse $C^\infty$ werden auch
\begin{definition}\label{def:8.5}\xindex{Fläche!reguläre}\xindex{Parametrisierung!reguläre}%
$S \subseteq \mdr^3$ heißt \textbf{reguläre Fläche} $:\gdw$
$\forall s \in S\;\exists $ Umgebung $V(s) \subseteq \mdr^3$ $\exists U \subseteq \mdr^2$ offen:
$\exists \text{ differenzierbare Abbildung } F: U \rightarrow V \cap S$:
$\forall s \in S\;\exists $ Umgebung $V(s) \subseteq \mdr^3$ $\exists U \subseteq \mdr^2$ offen:
$\exists \text{ differenzierbare Abbildung } F: U \rightarrow V \cap S$:
$\text{Rg}(J_F(u)) = 2\;\;\;\forall u \in U$.
$F$ heißt (lokale) \textbf{reguläre Parametrisierung} von $S$.
@ -440,7 +440,7 @@ Differenzierbare Mannigfaltigkeiten der Klasse $C^\infty$ werden auch
%\caption{}
\end{figure}
\[J_F(u,v) =
\[J_F(u,v) =
\begin{pmatrix}
-r(v) \sin u & r'(v) \cos u\\
r(v) \cos u & r'(v) \sin u\\
@ -449,7 +449,7 @@ Differenzierbare Mannigfaltigkeiten der Klasse $C^\infty$ werden auch
hat Rang 2 für alle $(u,v) \in \mdr^2$.
\item Kugelkoordinaten: $F: \mdr^2 \rightarrow \mdr^3$,\\
$(u, v) \mapsto (R \cos v \cos u, R \cos v \sin u, R \sin v)$\\
Es gilt: $F(u,v) \in S_R^2$, denn
Es gilt: $F(u,v) \in S_R^2$, denn
\begin{align*}
& R^2 \cos^2(v) \cos^2(u) + R^2 \cos^2(v) \sin^2(u) + R^2 \sin^2(v)\\
=& R^2 (\cos^2(v) \cos^2(u) + \cos^2(v) \sin^2(u) + \sin^2(v))\\
@ -459,7 +459,7 @@ Differenzierbare Mannigfaltigkeiten der Klasse $C^\infty$ werden auch
\end{align*}
Die Jacobi-Matrix
\[J_F(u,v) =
\[J_F(u,v) =
\begin{pmatrix}
-R \cos v \sin u & -R \sin v \cos u\\
R \cos v \cos u & -R \sin v \sin u\\
@ -480,10 +480,10 @@ Differenzierbare Mannigfaltigkeiten der Klasse $C^\infty$ werden auch
\begin{beweis}\leavevmode
$S \subseteq \mdr^3$ ist als reguläre Fläche eine 2-dimensionale Mannigfaltigkeit.
$S \subseteq \mdr^3$ ist als reguläre Fläche eine 2-dimensionale Mannigfaltigkeit.
Aus der Definition von regulären Flächen folgt direkt, dass Karten $(U_i, F_i)$ und
$(U_j \subseteq \mdr^2, F_j:\mdr^2 \rightarrow \mdr^3)$ von $S$ mit
$U_i \cap U_j \neq \emptyset$ existieren, wobei $F_i$ und $F_j$ nach
$U_i \cap U_j \neq \emptyset$ existieren, wobei $F_i$ und $F_j$ nach
Definition differenzierbare Abbildungen sind.
\underline{z.Z.:} $F_j^{-1} \circ F_i$ ist ein Diffeomorphismus.
@ -494,15 +494,15 @@ Differenzierbare Mannigfaltigkeiten der Klasse $C^\infty$ werden auch
\caption{Reguläre Fläche $S$ zum Beweis von \cref{kor:regular-surface-mannigfaltigkeit}}
\label{fig:parametric-surface-mapping}
\end{figure}
\underline{Idee:} Finde differenzierbare Funktion $\widetilde{F_j^{-1}}$
in Umgebung $W$ von $s$, sodass $\widetilde{F_j^{-1}}|_{S \cap W} = F_j^{-1}$.
\underline{Ausführung:} Sei $u_0 \in U_i$, $v_0 \in U_j$ mit $F_i(u_0) = s = F_j(v_0)$.
Da $\rang(J_{F_j}(v_0)) = 2$ ist, ist \obda
\[\det
Da $\rang(J_{F_j}(v_0)) = 2$ ist, ist \obda
\[\det
\begin{pmatrix}
\frac{\partial x}{\partial u} & \frac{\partial x}{\partial v}\\
\frac{\partial y}{\partial u} & \frac{\partial y}{\partial v}
@ -513,10 +513,10 @@ Differenzierbare Mannigfaltigkeiten der Klasse $C^\infty$ werden auch
Definiere $\widetilde{F_j}: U_j \times \mdr \rightarrow \mdr^3$ durch
\[\widetilde{F_j} (u, v, t) := \left(x(u,v), y(u,v), z(u,v)+t \right )\]
Offensichtlich: $\widetilde{F_j} |_{U_j \times \Set{0}} = F_j$
\[J_{\widetilde{F_j}} =
\[J_{\widetilde{F_j}} =
\begin{pmatrix}
\frac{\partial x}{\partial u} & \frac{\partial x}{\partial v} & 0\\
\frac{\partial y}{\partial u} & \frac{\partial y}{\partial v} & 0\\
@ -553,7 +553,7 @@ Differenzierbare Mannigfaltigkeiten der Klasse $C^\infty$ werden auch
\begin{beispiel}[Lie-Gruppen]
\begin{bspenum}
\item Alle endlichen Gruppen sind 0-dimensionale Lie-Gruppen.
\item $\GL_n(\mdr)$
\item $\GL_n(\mdr)$
% ist eine Lie-Gruppe, da sie nach \cref{bsp:gln-ist-mf} eine Mannigfaltigkeit ist.
% $\det: \GL_n \rightarrow \mdr$ ist eine stetige Abbildung.
\item $(\mdr^\times, \cdot)$
@ -653,10 +653,10 @@ Differenzierbare Mannigfaltigkeiten der Klasse $C^\infty$ werden auch
\begin{enumerate}[label=(\roman*),ref=\theenumii.\roman*]
\item Für $\Delta \in K$ und $S \subseteq \Delta$ Teilsimplex
ist $S \in K$.
\item \label{def:simplizialkomplex.ii} Für $\Delta_1, \Delta_2 \in K$ ist
$\Delta_1 \cap \Delta_2$ leer oder ein
Teilsimplex von $\Delta_1$ und von
$\Delta_2$.
\item \label{def:simplizialkomplex.ii} Für $\Delta_1, \Delta_2 \in K$ ist
$\Delta_1 \cap \Delta_2$ leer oder ein
Teilsimplex von $\Delta_1$ und von
$\Delta_2$.
\end{enumerate}
\item $|K| := \bigcup_{\Delta \in K} \Delta$ (mit Teilraumtopologie)
heißt \textbf{geometrische Realisierung}\xindex{Realisierung!geometrische}
@ -723,7 +723,7 @@ Differenzierbare Mannigfaltigkeiten der Klasse $C^\infty$ werden auch
\input{figures/topology-linear-mapping.tex}
\item Folgende Abbildung $\varphi: \Delta^n \rightarrow \Delta^{n-1}$
\item Folgende Abbildung $\varphi: \Delta^n \rightarrow \Delta^{n-1}$
ist simplizial:
\input{figures/topology-triangle-to-line.tex}
@ -742,7 +742,7 @@ Differenzierbare Mannigfaltigkeiten der Klasse $C^\infty$ werden auch
Sei $K$ ein endlicher Simplizialkomplex. Für $n \geq 0$ sei
$a_n(K)$ die Anzahl der $n$-Simplizes in $K$.
Dann heißt
Dann heißt
\[\chi(K) := \sum_{n=0}^{\dim K} (-1)^n a_n(K)\]
\textbf{Eulerzahl} (oder Euler-Charakteristik\index{Euler-Charakteristik|see{Eulerzahl}})
von $K$.
@ -842,7 +842,7 @@ Differenzierbare Mannigfaltigkeiten der Klasse $C^\infty$ werden auch
\begin{bemerkung}\label{kor:simplex-unterteilung}
Sei $\Delta$ ein $n$-Simplex und $x \in \Delta^\circ \subseteq \mdr^n$.
Sei $K$ der Simplizialkomplex, der aus $\Delta$ durch
Sei $K$ der Simplizialkomplex, der aus $\Delta$ durch
\enquote{Unterteilung} in $x$ entsteht. Dann ist $\chi(K) = \chi(\Delta) = 1$.
\end{bemerkung}
@ -920,7 +920,7 @@ Differenzierbare Mannigfaltigkeiten der Klasse $C^\infty$ werden auch
$\partial P$ von $0$ aus auf $\partial \fB_1(0) = S^2$.
Erhalte Triangulierung von $S^2$.
\item Sind $P_1$ und $P_2$ konvexe Polygone und $T_1, T_2$
die zugehörigen Triangulierungen von $S^2$, so gibt es
die zugehörigen Triangulierungen von $S^2$, so gibt es
eine Triangulierung $T$, die sowohl um $T_1$ als
auch um $T_2$ Verfeinerung ist (vgl. \cref{fig:topology-3}).
@ -947,7 +947,7 @@ Differenzierbare Mannigfaltigkeiten der Klasse $C^\infty$ werden auch
und $C_n(K)$ der $\mdr$-Vektorraum mit Basis $A_n(K)$, d.~h.
\[C_n(K) = \Set{\sum_{\sigma \in A_n(K)} c_\sigma \cdot \sigma | c_\sigma \in \mdr}\]
Sei $\sigma = \Delta(x_0, \dots, x_n) \in A_n(K)$, sodass
Sei $\sigma = \Delta(x_0, \dots, x_n) \in A_n(K)$, sodass
$x_0 < x_1 < \dots < x_n$.
Für $i = 0, \dots, n$ sei $\partial_i \sigma := \Delta(x_0, \dots, \hat{x_i}, \dots, x_n)$
@ -1001,14 +1001,14 @@ Differenzierbare Mannigfaltigkeiten der Klasse $C^\infty$ werden auch
\end{beweis}
\begin{definition}%
Sei $K$ ein Simplizialkomplex,
$Z_n := \text{Kern}(d_n) \subseteq C_n$ und
Sei $K$ ein Simplizialkomplex,
$Z_n := \text{Kern}(d_n) \subseteq C_n$ und
$B_n := \text{Bild}(d_{n+1}) \subseteq C_n$.
\begin{defenum}
\item $H_n = H_n(K, \mdr) := Z_n / B_n$ heißt $n$-te
\item $H_n = H_n(K, \mdr) := Z_n / B_n$ heißt $n$-te
\textbf{Homologiegruppe}\xindex{Homologiegruppe} von $K$.
\item $b_n(K) := \dim_{\mdr} H_n$ heißt $n$-te
\item $b_n(K) := \dim_{\mdr} H_n$ heißt $n$-te
\textbf{Betti-Zahl}\xindex{Betti-Zahl} von $K$.
\end{defenum}
\end{definition}

View file

@ -6,7 +6,7 @@
\section{Homotopie von Wegen}
\begin{figure}[ht]
\centering
\subfloat[$\gamma_1$ und $\gamma_2$ sind homotop, da man sie
\subfloat[$\gamma_1$ und $\gamma_2$ sind homotop, da man sie
\enquote{zueinander verschieben} kann.]{
\input{figures/topology-homotop-paths.tex}
\label{fig:homotope-wege-anschaulich}
@ -20,7 +20,7 @@
\end{figure}
\begin{definition}%
Sei $X$ ein topologischer Raum, $a, b \in X$,
Sei $X$ ein topologischer Raum, $a, b \in X$,
$\gamma_1, \gamma_2: I \rightarrow X$ Wege von $a$ nach $b$,
d.~h. $\gamma_1(0) = \gamma_2(0) = a$, $\gamma_1(1) = \gamma_2(1) = b$
@ -38,7 +38,7 @@
\end{definition}
\begin{bemerkung}
Sei $X$ ein topologischer Raum, $a, b \in X$,
Sei $X$ ein topologischer Raum, $a, b \in X$,
$\gamma_1, \gamma_2: I \rightarrow X$ Wege von $a$ nach $b$
und $H$ eine Homotopie zwischen $\gamma_1$ und $\gamma_2$.
@ -70,7 +70,7 @@
H'(t, 2s) &\text{falls } 0 \leq s \leq \frac{1}{2}\\
H''(t, 2s-1) &\text{falls } \frac{1}{2} \leq s \leq 1\end{cases}$
$\Rightarrow$ $H$ ist stetig und Homotopie von $\gamma_1$ nach
$\Rightarrow$ $H$ ist stetig und Homotopie von $\gamma_1$ nach
$\gamma_3$.
\end{itemize}
$\qed$
@ -78,12 +78,12 @@
\begin{beispiel}
\begin{bspenum}
\item Sei $X = S^1$. $\gamma_1$ und $\gamma_2$ aus
\item Sei $X = S^1$. $\gamma_1$ und $\gamma_2$ aus
\cref{fig:circle-two-paths} nicht homotop.
\item Sei $X = T^2$. $\gamma_1, \gamma_2$ und $\gamma_3$
aus \cref{fig:torus-three-paths} sind paarweise
nicht homotop.
\item Sei $X = \mdr^2$ und $a=b=(0,0)$.
\item Sei $X = \mdr^2$ und $a=b=(0,0)$.
Je zwei Wege im $\mdr^2$ mit Anfangs- und Endpunkt $(0,0)$
sind homotop.
@ -99,7 +99,7 @@
$\gamma_0(t) = (0,0) \; \forall t \in I$. Sei
$\gamma(0) = \gamma(1) = (0,0)$.
$H(t,s) := (1-s) \gamma(t)$ ist stetig,
$H(t,s) := (1-s) \gamma(t)$ ist stetig,
$H(t,0) = \gamma(t)\; \forall t \in I$ und
$H(t,1) = (0,0) \; \forall t \in I$.
\end{bspenum}
@ -123,7 +123,7 @@
% Mitschrieb vom 05.12.2013 %
%%%%%%%%%%%%%%%%%%%%%%%%%%%%%%%%%%%%%%%%%%%%%%%%%%%%%%%%%%%%%%%%%%%%%
\begin{bemerkung}\label{kor:homotope-wege}
Sei $X$ ein topologischer Raum, $\gamma: I \rightarrow X$ ein
Sei $X$ ein topologischer Raum, $\gamma: I \rightarrow X$ ein
Weg und $\varphi: I \rightarrow I$ stetig mit $\varphi(0) = 0$,
$\varphi(1) = 1$. Dann sind $\gamma$ und $\gamma \circ \varphi$
homotop.
@ -139,7 +139,7 @@
\begin{definition}\xindex{Weg!zusammengesetzter}%
Seien $\gamma_1, \gamma_2$ Wege in $X$ mit $\gamma_1(1) = \gamma_2(0)$.
Dann ist
Dann ist
\[\gamma (t) = \begin{cases}
\gamma_1(2t) &\text{falls } 0 \leq t < \frac{1}{2}\\
\gamma_2(2t-1) &\text{falls } \frac{1}{2} \leq t \leq 1
@ -149,7 +149,7 @@
\end{definition}
\begin{bemerkung}\label{kor:assoziativitaet-von-zusammensetzen-von-wegen}
Das Zusammensetzen von Wegen ist nur bis auf
Das Zusammensetzen von Wegen ist nur bis auf
Homotopie assoziativ, d.~h.:
\begin{align*}
\gamma_1 * (\gamma_2 * \gamma_3) &\neq (\gamma_1 * \gamma_2) * \gamma_3\\
@ -203,13 +203,13 @@
Sei $H_i$ eine Homotopie zwischen $\gamma_i$ und $\gamma_i'$,
$i=1,2$.
Dann ist
Dann ist
\[H(t,s) := \begin{cases}
H_1(2t, s) &\text{falls } 0 \leq t \leq \frac{1}{2}\;\;\;\forall s \in I\\
H_2(2t-1,s) &\text{falls } \frac{1}{2} \leq t \leq 1
\end{cases}\]
eine Homotopie zwischen
eine Homotopie zwischen
$\gamma_1 * \gamma_2$ und $\gamma_1' * \gamma_2 '$.
\end{beweis}
@ -219,7 +219,7 @@ Eine spezielle Homotopieäquivalenz sind sog. Deformationsretraktionen:
und $\iota = (\id_X)|_A$.
\begin{defenum}
\item $\iota: A \rightarrow X$ mit $\iota(x) = x$ heißt die
\item $\iota: A \rightarrow X$ mit $\iota(x) = x$ heißt die
\textbf{Inklusionsabbildung}\xindex{Inklusionsabbildung} und
man schreibt: $\iota: A \hookrightarrow X$.
\item $r$ heißt \textbf{Retraktion}\xindex{Retraktion}, wenn $r|_A = \id_A$ ist.
@ -264,7 +264,7 @@ Für einen Weg $\gamma$ sei $[\gamma]$ seine \textbf{Homotopieklasse}\xindex{Hom
\item Assoziativität folgt aus \cref{kor:assoziativitaet-von-zusammensetzen-von-wegen}
\item Neutrales Element $e = [\gamma_0], \gamma_0(t) = x \;\;\; \forall t \in I$.
$e * [\gamma] = [\gamma] = [\gamma] * e$, da $\gamma_0 * \gamma \sim \gamma$
\item \xindex{Weg!inverser} Inverses Element $[\gamma]^{-1} = [\overline{\gamma}] = [\gamma(1-t)]$,
\item \xindex{Weg!inverser} Inverses Element $[\gamma]^{-1} = [\overline{\gamma}] = [\gamma(1-t)]$,
denn $\overline{\gamma} * \gamma \sim \gamma_0 \sim \gamma * \overline{\gamma}$
\end{enumerate}
\end{beweis}
@ -280,7 +280,7 @@ Für einen Weg $\gamma$ sei $[\gamma]$ seine \textbf{Homotopieklasse}\xindex{Hom
$[\gamma^k] \mapsto k$ ist ein Isomorphismus.
\item $\pi_1 (\mdr^2, 0) = \pi_1 (\mdr^2, x) = \Set{e}$ für jedes $x \in \mdr^2$
\item $\pi_1 (\mdr^n, x) = \Set{e}$ für jedes $x \in \mdr^n$
\item $G \subseteq \mdr^n$ heißt \textbf{sternförmig}\xindex{sternförmig} bzgl. $x \in G$,
\item $G \subseteq \mdr^n$ heißt \textbf{sternförmig}\xindex{sternförmig} bzgl. $x \in G$,
wenn für jedes $y \in G$ auch die Strecke $[x, y] \subseteq G$
ist.
@ -338,7 +338,7 @@ Für einen Weg $\gamma$ sei $[\gamma]$ seine \textbf{Homotopieklasse}\xindex{Hom
für ein $x \in X$.
\end{definition}
Wenn $\pi_1(X,x) = \Set{e}$ für ein $x \in X$ gilt, dann wegen
Wenn $\pi_1(X,x) = \Set{e}$ für ein $x \in X$ gilt, dann wegen
\cref{kor:gruppenisomorphismus-wege} sogar für alle $x \in X$.
\begin{bemerkung}\label{korr:11.5}
@ -359,7 +359,7 @@ Wenn $\pi_1(X,x) = \Set{e}$ für ein $x \in X$ gilt, dann wegen
\item $f_*$ ist wohldefiniert: Seien $\gamma_1, \gamma_2$ homotope
Wege von $x$. z.Z.: $f \circ \gamma_1 \sim f \circ \gamma_2$:
Nach Voraussetzung gibt es stetige Abbildungen $H:I\times I \rightarrow X$
mit
mit
\begin{align*}
H(t,0) &= \gamma_1(t),\\
H(t,1) &= \gamma_2(t),\\
@ -376,7 +376,7 @@ Wenn $\pi_1(X,x) = \Set{e}$ für ein $x \in X$ gilt, dann wegen
\begin{beispiel}
\begin{bspenum}
\item $f:S^1 \hookrightarrow \mdr^2$ ist injektiv, aber
\item $f:S^1 \hookrightarrow \mdr^2$ ist injektiv, aber
$f_*:\pi_1(S^1, 1) \cong \mdz \rightarrow \pi_1(\mdr^2, 1) = \Set{e}$
ist nicht injektiv.
\item $f: \mdr \rightarrow S^1, t \mapsto (\cos 2 \pi t, \sin 2 \pi t)$
@ -406,7 +406,7 @@ Wenn $\pi_1(X,x) = \Set{e}$ für ein $x \in X$ gilt, dann wegen
stetig mit $f(x_0) = y_0 = g(x_0)$.
$f$ und $g$ heißen \textbf{homotop} ($f \sim g$), wenn es eine stetige
Abbildung $H: X \times I \rightarrow Y$ mit
Abbildung $H: X \times I \rightarrow Y$ mit
\begin{align*}
H(x,0) &= f(x) \; \forall x \in X\\
H(x,1) &= g(x) \; \forall x \in X\\
@ -426,7 +426,7 @@ Wenn $\pi_1(X,x) = \Set{e}$ für ein $x \in X$ gilt, dann wegen
Z.~z.: $f \circ \gamma \sim g \circ \gamma$
Sei dazu $H_\gamma: I \times I \rightarrow Y, (t,s) \mapsto H(\gamma(t), s)$.
Dann gilt:
Dann gilt:
\begin{align*}
H_\gamma(t,0) &= H(\gamma(t), 0) = (f \circ \gamma)(t) \;\forall t \in I\\
H_\gamma(1,s) &= H(\gamma(1), s) = H(x_0, s) = y_0\;\forall s \in I\\
@ -450,7 +450,7 @@ Wenn $\pi_1(X,x) = \Set{e}$ für ein $x \in X$ gilt, dann wegen
\end{beispiel}
\begin{satz}[Satz von Seifert und van Kampen \enquote{light}]\label{thm:seifert-van-kampen}
Sei $X$ ein topologischer Raum, $U, V \subseteq X$ offen mit
Sei $X$ ein topologischer Raum, $U, V \subseteq X$ offen mit
$U \cup V = X$ und $U \cap V$ wegzusammenhängend.
Dann wird $\pi_1(X,x)$ für $x \in U \cap V$ erzeugt von geschlossenen
@ -460,14 +460,14 @@ Wenn $\pi_1(X,x) = \Set{e}$ für ein $x \in X$ gilt, dann wegen
\begin{beweis}
Sei $\gamma: I \rightarrow X$ ein geschlossener Weg um $x$.
Überdecke $I$ mit endlich vielen offenen Intervallen
$I_1, I_2, \dots, I_n$, die ganz in
$I_1, I_2, \dots, I_n$, die ganz in
$\gamma^{-1}(U)$ oder ganz in $\gamma^{-1}(V)$ liegen.
\Obda sei $\gamma(I_1) \subseteq U, \gamma(I_2) \subseteq V$, etc.
Wähle $t_i \in I_i \cap I_{i+1}$, also $\gamma(t_i) \in U \cap V$.
Sei $\sigma_i$ Weg in $U \cap V$ von $x_0$ nach $\gamma(t_i) \Rightarrow \gamma$
ist homotop zu
ist homotop zu
\[\underbrace{\gamma_1 * \overline{\sigma_1}}_{\text{in } U} * \underbrace{\sigma_1 * \gamma_2 * \overline{\sigma_2}}_{\text{in } V} * \dots * \sigma_{n-1} * \gamma_2 \text{ mit } \gamma_i := \gamma |_{I_i}\]
\end{beweis}
@ -547,17 +547,17 @@ Wenn $\pi_1(X,x) = \Set{e}$ für ein $x \in X$ gilt, dann wegen
\begin{beweis}
Sei $p: Y \rightarrow X$ eine Überlagerung und $x \in X$ beliebig.
Dann existiert eine offene Umgebung $U(x) \subseteq X$ und offene
Teilmengen $V_j \subseteq X$ mit
Teilmengen $V_j \subseteq X$ mit
$p^{-1}(U) = \Dcup V_j$ und
$p|_{V_j}: V_j \rightarrow U$ ist Homöomorphismus.
D.~h. es existiert ein $y \in V_j$, so dass $p|_{V_j}(y) = x$.
Da $x \in X$ beliebig war und ein $y \in Y$ existiert, mit
Da $x \in X$ beliebig war und ein $y \in Y$ existiert, mit
$p(y) = x$, ist $p$ surjektiv. $\qed$
\end{beweis}
\begin{definition}\xindex{Abbildung!offene}%
Seien $(X, \fT_X), (Y, \fT_Y)$ topologische Räume und $f:X \rightarrow Y$ eine
Seien $(X, \fT_X), (Y, \fT_Y)$ topologische Räume und $f:X \rightarrow Y$ eine
Abbildung.
$f$ heißt \textbf{offen} $:\gdw \forall U \in \fT_X: f(U) \in \fT_Y$.
@ -600,7 +600,7 @@ Wenn $\pi_1(X,x) = \Set{e}$ für ein $x \in X$ gilt, dann wegen
\begin{definition}\xindex{diskret}%
Sei $X$ ein topologischer Raum und $M \subseteq X$.
$M$ heißt \textbf{diskret} in $X$, wenn $M$ in $X$ keinen
$M$ heißt \textbf{diskret} in $X$, wenn $M$ in $X$ keinen
Häufungspunkt hat.
\end{definition}
@ -628,11 +628,11 @@ Wenn $\pi_1(X,x) = \Set{e}$ für ein $x \in X$ gilt, dann wegen
$\Rightarrow V_{j_1} \cap V_{j_2} = \emptyset$ nach Voraussetzung.
\underline{2. Fall}: $p(y_1) \neq p(y_2)$.
Dann seien $U_1$ und $U_2$ disjunkte Umgebungen von $p(y_1)$
und $p(y_2)$.
$\Rightarrow p^{-1}(U_1)$ und $p^{-1}(U_2)$ sind disjunkte
$\Rightarrow p^{-1}(U_1)$ und $p^{-1}(U_2)$ sind disjunkte
Umgebungen von $y_1$ und $y_2$.
\item Sei $x \in X$ beliebig, aber fest.
@ -717,7 +717,7 @@ Wenn $\pi_1(X,x) = \Set{e}$ für ein $x \in X$ gilt, dann wegen
Sei $q:U \rightarrow V$ die Umkehrabbildung zu $p|_V$.
Sei $W:= f^{-1}(U) \cap f_0^{-1}(V) \cap f_1^{-1}(V)$. $W$ ist
Sei $W:= f^{-1}(U) \cap f_0^{-1}(V) \cap f_1^{-1}(V)$. $W$ ist
offene Umgebung in $Z$ von $z$.
\underline{Behauptung:} $W \subseteq T$
@ -749,7 +749,7 @@ $p|_{V_j}: V_j \rightarrow U$ Homöomorphismus.
\begin{bemerkung}%Bemerkung 12.6 der Vorlesung
Wege in $X$ lassen sich zu Wegen in $Y$ liften.
Zu jedem $y \in p^{-1}(\gamma(0))$ gibt es genau einen Lift von
Zu jedem $y \in p^{-1}(\gamma(0))$ gibt es genau einen Lift von
$\gamma$.
\end{bemerkung}
@ -757,7 +757,7 @@ $p|_{V_j}: V_j \rightarrow U$ Homöomorphismus.
Seien $p: Y \rightarrow X$ eine Überlagerung, $a,b \in X$,
$\gamma_0, \gamma_1: I \rightarrow X$ homotope Wege von $a$ nach
$b$, $\tilde{a} \in p^{-1}(a), \tilde{\gamma_0}, \tilde{\gamma_1}$
Liftungen von $\gamma_0$ bzw. $\gamma_1$ mit
Liftungen von $\gamma_0$ bzw. $\gamma_1$ mit
$\tilde{\gamma_i}(0) = \tilde{a}$.
Dann ist $\tilde{\gamma_0}(1) = \tilde{\gamma_1}(1)$ und
@ -784,7 +784,7 @@ $p|_{V_j}: V_j \rightarrow U$ Homöomorphismus.
Da $p^{-1}(b)$ diskrete Teilmenge von $Y$ ist\\
$\Rightarrow \tilde{b_s} = \tilde{H}(1,s) = \tilde{H}(1,0) \;\forall s \in I$\\
$\Rightarrow \tilde{b_0} = \tilde{b_1}$ und $\tilde{H}$ ist Homotopie
$\Rightarrow \tilde{b_0} = \tilde{b_1}$ und $\tilde{H}$ ist Homotopie
zwischen $\tilde{\gamma_0}$ und $\tilde{\gamma_1}$. $\qed$
\end{beweis}
@ -801,7 +801,7 @@ $p|_{V_j}: V_j \rightarrow U$ Homöomorphismus.
\item Sei $\tilde{\gamma}$ ein Weg in $Y$ um $y_0$ und
$p_* ([\tilde{\gamma}]) = e$, also $p \circ \tilde{\gamma} \sim \gamma_{x_0}$
Nach \cref{proposition:12.7} ist dann
Nach \cref{proposition:12.7} ist dann
$\tilde{\gamma}$ homotop zum Lift des konstanten Wegs
$\gamma_{x_0}$ mit Anfangspunkt $y_0$, also zu
$\gamma_{y_0} \Rightarrow [\tilde{\gamma}] = e$
@ -823,9 +823,9 @@ $p|_{V_j}: V_j \rightarrow U$ Homöomorphismus.
Zu $i \in \Set{0, \dots, d-1}$ gibt es Weg $\delta_i$ in
$Y$ mit $\delta_i(0) = y_0$ und $\delta_i(1) = y_i$\\
$\Rightarrow p \cup \delta_i$ ist geschlossener Weg in
$\Rightarrow p \cup \delta_i$ ist geschlossener Weg in
$X$ um $x_0$.\\
$\Rightarrow$ Jedes $y_i$ mit $i=0, \dots, d-1$ ist
$\Rightarrow$ Jedes $y_i$ mit $i=0, \dots, d-1$ ist
$\tilde{\gamma}(1)$ für ein $[\gamma] \in \pi_1(X,x_0)$.
\end{enumerate}
\end{beweis}
@ -885,7 +885,7 @@ $p|_{V_j}: V_j \rightarrow U$ Homöomorphismus.
Offensichtlich ist $q(\tilde{p}(z)) = p(z)$.
\underline{Zu zeigen:} $\tilde{p}$ ist stetig in $z \in \tilde{X}$:
Sei $W \subseteq Y$ offene Umgebung von $\tilde{p}(z)$.
$\xRightarrow{q \text{ offen}} q(W)$ ist offene Umgebung von $p(z) \cdot d(\tilde{p}(z))$.
@ -915,8 +915,8 @@ $p|_{V_j}: V_j \rightarrow U$ Homöomorphismus.
\end{folgerung}
\begin{beweis}
Seien $x_0 \in X, \tilde{x_0} \in \tilde{X}$ mit
$p(\tilde{x_0}) = x_0$ und
Seien $x_0 \in X, \tilde{x_0} \in \tilde{X}$ mit
$p(\tilde{x_0}) = x_0$ und
$\tilde{y_0} \in q^{-1}(x_0) \subseteq \tilde{Y}$.
Nach \cref{thm:12.11} gibt es genau eine Überlagerung
@ -925,7 +925,7 @@ $p|_{V_j}: V_j \rightarrow U$ Homöomorphismus.
\[g: \tilde{Y} \rightarrow \tilde{X} \text{ mit } g(\tilde{y_0}) = \tilde{x_0} \text{ und } p \circ g = q\]
Damit gilt: $p \circ q \circ f = q \circ f = p$, $q \circ f \circ g = p \circ g = q$.
Also ist $g \circ f: \tilde{X} \rightarrow \tilde{X}$ Lift von
Also ist $g \circ f: \tilde{X} \rightarrow \tilde{X}$ Lift von
$p:\tilde{X} \rightarrow X$ mit $(g \circ f) (\tilde{x_0}) = \tilde{x_0}$.
Da auch $\id_{\tilde{x}}$ diese Eigenschaft hat, folgt mit
@ -961,7 +961,7 @@ der folgende Satz:
\[\tilde{U} = \tilde{U}(x, [\gamma]) := \Set{(y, [\gamma * \alpha]) | y \in U, \alpha \text{ Weg in } U \text{ von } x \text{ nach } y} \]
$p$ ist Überlagerung: $p|_{\tilde{U}} : \tilde{U} \rightarrow U$
bijektiv. $p$ ist stetig und damit $p|_{\tilde{U}}$ ein
bijektiv. $p$ ist stetig und damit $p|_{\tilde{U}}$ ein
Homöomorphismus.
Sind $\gamma_1, \gamma_2$ Wege von $x_0$ nach $x$ und $\gamma_1 \sim \gamma_2$,
@ -990,7 +990,7 @@ der folgende Satz:
\begin{defenum}
\item $f$ heißt \textbf{Decktransformation} von $p :\gdw p \circ f = p$.
\item Die Decktransformationen von $p: Y \rightarrow X$ bilden mit der Verkettung eine Gruppe,
\item Die Decktransformationen von $p: Y \rightarrow X$ bilden mit der Verkettung eine Gruppe,
die sog. \textbf{Decktransformationsgruppe}\xindex{Decktransformationsgruppe}.
Man schreibt:
$\Deck(p)$, $\Deck(Y/X)$ oder $\Deck(Y \rightarrow X)$.
@ -1023,10 +1023,10 @@ der folgende Satz:
\end{itemize}
\item Die Menge
\[\Fix(f) = \Set{y \in Y | f(y) = y}\]
ist abgeschlossen als Urbild der Diagonale
ist abgeschlossen als Urbild der Diagonale
$\Delta \subseteq Y \times Y$ unter der stetigen
Abbildung $y \mapsto (f(y),y)$. Außerdem ist $\Fix(f)$
offen, denn ist $y \in \Fix(f)$, so sei $U$ eine
offen, denn ist $y \in \Fix(f)$, so sei $U$ eine
Umgebung von $p(y) \in X$ wie in \cref{def:12.1}
und $U \subseteq p^{-1}(U)$ die Komponente, die $y$
enthält; also $p:V \rightarrow U$ ein Homöomorphismus.
@ -1041,7 +1041,7 @@ der folgende Satz:
\item Es sei $x_0 \in X$, $\deg(p) = d$ und $p^{-1}(x_0) = \Set{y_0, \dots, y_{d-1}}$.
Für $f \in \Deck(Y/X)$ ist $f(y_0)= \Set{y_0, \dots, y_{d-1}}$.
Zu $i \in \Set{0, \dots, d-1}$ gibt es höchstens ein
Zu $i \in \Set{0, \dots, d-1}$ gibt es höchstens ein
$f \in \Deck(Y/X)$ mit $f(y_0) = y_1$, denn ist
$f(y_0) = g(y_0)$, so ist $(g^{-1} \circ f)(y_0) = y_0$,
also nach \cref{kor:12.14c} $g^{-1} \circ f = \id_Y$.
@ -1061,7 +1061,7 @@ Nun werden wir eine Verbindung zwischen der Decktransformationsgruppe
und der Fundamentalgruppe herstellen:
\begin{satz}\label{thm:12.15}%In Vorlesung: Satz 12.15
Ist $p: \tilde{X} \rightarrow X$ eine universelle Überlagerung,
Ist $p: \tilde{X} \rightarrow X$ eine universelle Überlagerung,
so gilt:
\[\Deck(\tilde{X}/X) \cong \pi_1(X, x_0)\;\;\;\forall x_0 \in X\]
\end{satz}
@ -1074,11 +1074,11 @@ und der Fundamentalgruppe herstellen:
eindeutig bestimmt und damit auch $\rho$ wohldefiniert.
\begin{itemize}
\item \underline{$\rho$ ist Gruppenhomomorphismus}: Seien
\item \underline{$\rho$ ist Gruppenhomomorphismus}: Seien
$f, g \in \Deck(\tilde{X}/ X) \Rightarrow \gamma_{g \circ f} = \gamma_g * g(\gamma_f)$
$\Rightarrow p(\gamma_{g \circ f}) = p(\gamma_g) * \underbrace{(p \circ g)}_{=p} (\gamma_f) = \rho(g) \neq \rho(f)$
\item \underline{$\rho$ ist injektiv}: $\rho(f) = e \Rightarrow p (\gamma_f) \sim \gamma_{x_0}$
$\xRightarrow{\cref{thm:ueberlagerung-weg-satz-12.6}} \gamma_f \sim \gamma_{\tilde{x_0}}$
$\xRightarrow{\cref{thm:ueberlagerung-weg-satz-12.6}} \gamma_f \sim \gamma_{\tilde{x_0}}$
$\Rightarrow f(x_0) = \tilde{x_0} \xRightarrow{\crefabbr{kor:12.14c}} f = \id_{\tilde{x}}$.
\item \underline{$\rho$ ist surjektiv}: Sei $[\gamma] \in \pi_1(X, x_0)$,
$\tilde{\gamma}$ Lift von $\gamma$ nach $\tilde{x}$ mit
@ -1086,7 +1086,7 @@ und der Fundamentalgruppe herstellen:
sei $\tilde{x_1}$.
\underline{$p$ ist reguläre Überlagerung}: Seien
$\tilde{x_0}, \tilde{x_1} \in \tilde{X}$ mit
$\tilde{x_0}, \tilde{x_1} \in \tilde{X}$ mit
$p(\tilde{x_0}) = p(\tilde{x_1})$. Nach \cref{thm:12.11}
gibt es genau eine Überlagerung $\tilde{p}: \tilde{X} \rightarrow X$
mit $p=p \circ \tilde{p}$ und $\tilde{p}(\tilde{x_0}) = \tilde{x_1}$.
@ -1163,7 +1163,7 @@ und der Fundamentalgruppe herstellen:
\[m_g: X \rightarrow X, x \mapsto g \circ x\]
ein Homöomorphismus ist.
\item Ist $G$ eine topologische Gruppe, so heißt die Gruppenoperation $\circ$
\textbf{stetig}\xindex{Gruppenoperation!stetige}, wenn
\textbf{stetig}\xindex{Gruppenoperation!stetige}, wenn
\[\forall g \in G: m_g \text{ ist stetig}\]
gilt.
\end{defenum}
@ -1175,7 +1175,7 @@ und der Fundamentalgruppe herstellen:
\begin{beweis}\leavevmode
Nach Voraussetzung ist $m_g := \circ |_{\Set{g} \times X} : X \rightarrow X, x \mapsto g \circ x$ stetig.
Die Umkehrabbildung zu $m_g$ ist $m_{g^{-1}}$:
Die Umkehrabbildung zu $m_g$ ist $m_{g^{-1}}$:
\begin{align*}
(m_{g^{-1}} \circ m_g)(x) &= m_{g^{-1}} (m_g (x))\\
&= m_{g^{-1}} (g \circ x)\\
@ -1196,7 +1196,7 @@ und der Fundamentalgruppe herstellen:
\begin{bemenum}
\item Die Gruppenoperation von $G$ auf $X$ entsprechen bijektiv
den Gruppenhomomorphismen $\varrho: G \rightarrow \Perm(X) = \Sym(X) = \Set{f: X \rightarrow X | f \text{ ist bijektiv}}$
\item Ist $X$ ein topologischer Raum, so entsprechen dabei
\item Ist $X$ ein topologischer Raum, so entsprechen dabei
die Gruppenoperationen durch Homöomorphismus den Gruppenhomomorphismen
$G \rightarrow \Homoo(X)$
\end{bemenum}
@ -1213,7 +1213,7 @@ und der Fundamentalgruppe herstellen:
Umgekehrt: Sei $\varrho: G \rightarrow \Perm(X)$ Gruppenhomomorphismus. Definiere $\circ: G \times X \rightarrow X$ durch $g \circ x = \varrho (g)(x)$.
z.~z. \cref{def:gruppenoperation.2}:
z.~z. \cref{def:gruppenoperation.2}:
\begin{align*}
g_1 \circ (g_2 \circ x) &= \varrho (g_1) (g_2 \circ x)\\
&= \varrho(g_1) (\varrho(g_2)(x))\\
@ -1222,8 +1222,8 @@ und der Fundamentalgruppe herstellen:
&= (g_1 \cdot g_2) \circ x
\end{align*}
z.~z. \cref{def:gruppenoperation.1}:
$1_G \cdot x = \varrho(1_G)(x) = \id_X(x) = x$, weil $\varrho$ ein
z.~z. \cref{def:gruppenoperation.1}:
$1_G \cdot x = \varrho(1_G)(x) = \id_X(x) = x$, weil $\varrho$ ein
Homomorphismus ist.
\end{beweis}
@ -1253,7 +1253,7 @@ und der Fundamentalgruppe herstellen:
\begin{enumerate}[label=\roman*)]
\item $[e] \circ \tilde{x} = \rtilde{e * \delta} = \tilde{x}$
\item $\rtilde{\gamma_1 * \gamma_2 * \delta}(1) = [\gamma_1 * \gamma_2] \circ \tilde{x} = ([\gamma_1] * [\gamma_2]) \circ \tilde{x}$\\
$\gamma_1 * \gamma_2 * \delta(1) = [\gamma_1] \circ (\tilde{\gamma_2 * \delta})(1) = [\gamma_1] \circ ([\gamma_2] \circ \tilde{x})$
$\gamma_1 * \gamma_2 * \delta(1) = [\gamma_1] \circ (\tilde{\gamma_2 * \delta})(1) = [\gamma_1] \circ ([\gamma_2] \circ \tilde{x})$
\end{enumerate}
\end{beispiel}

View file

@ -18,7 +18,7 @@
\end{aufgabe}
\begin{aufgabe}\label{ub11:aufg3}
Sei $(X, d)$ eine absolute Ebene. Der \textit{Abstand}\xindex{Abstand} eines
Sei $(X, d)$ eine absolute Ebene. Der \textit{Abstand}\xindex{Abstand} eines
Punktes $P$ zu einer Menge $Y \subseteq X$ von Punkten ist
definiert durch $d(P, Y) := \inf{d(P, y) | y \in Y}$.
@ -27,13 +27,13 @@
\item \label{ub11:aufg3.a} Ist $\triangle ABC$ ein Dreieck, in dem die Seiten
$\overline{AB}$ und $\overline{AC}$ kongruent sind, so
sind die Winkel $\angle ABC$ und $\angle BCA$ gleich.
\item \label{ub11:aufg3.b} Ist $\triangle ABC$ ein beliebiges Dreieck, so liegt
\item \label{ub11:aufg3.b} Ist $\triangle ABC$ ein beliebiges Dreieck, so liegt
der längeren Seite der größere Winkel gegenüber und
umgekehrt.
\item \label{ub11:aufg3.c} Sind $g$ eine Gerade und $P \notin g$ ein Punkt, so gibt
es eine eindeutige Gerade $h$ mit $P \in h$ und die
$g$ im rechten Winkel schneidet. Diese Grade heißt
\textit{Lot}\xindex{Lot} von $P$ auf $g$ und der
$g$ im rechten Winkel schneidet. Diese Grade heißt
\textit{Lot}\xindex{Lot} von $P$ auf $g$ und der
Schnittpunkt des Lots mit $g$ heißt \textit{Lotfußpunkt}\xindex{Lotfußpunkt}.
\end{aufgabeenum}
\end{aufgabe}

View file

@ -12,25 +12,25 @@
\section{Axiome für die euklidische Ebene}
Axiome\xindex{Axiom} bilden die Grundbausteine jeder mathematischen Theorie. Eine
Sammlung aus Axiomen nennt man Axiomensystem\xindex{Axiomensystem}.
Da der Begriff des Axiomensystems so grundlegend ist, hat man auch
Da der Begriff des Axiomensystems so grundlegend ist, hat man auch
ein paar sehr grundlegende Forderungen an ihn: Axiomensysteme sollen
\textbf{widerspruchsfrei} sein, die Axiome sollen möglichst
\textbf{unabhängig} sein und \textbf{Vollständigkeit} wäre auch toll.
Mit Unabhängigkeit ist gemeint, dass kein Axiom sich aus einem anderem
herleiten lässt. Dies scheint auf den ersten Blick eine einfache
Eigenschaft zu sein. Auf den zweiten Blick muss man jedoch einsehen,
dass das Parallelenproblem, also die Frage ob das Parallelenaxiom
unabhängig von den restlichen Axiomen ist, über 2000 Jahre nicht
Eigenschaft zu sein. Auf den zweiten Blick muss man jedoch einsehen,
dass das Parallelenproblem, also die Frage ob das Parallelenaxiom
unabhängig von den restlichen Axiomen ist, über 2000 Jahre nicht
gelöst wurde. Ein ganz anderes Kaliber ist die Frage nach der
Vollständigkeit. Ein Axiomensystem gilt als Vollständig, wenn
jede Aussage innerhalb des Systems verifizierbar oder falsifizierbar
ist. Interessant ist hierbei der Gödelsche Unvollständigkeitssatz,
ist. Interessant ist hierbei der Gödelsche Unvollständigkeitssatz,
der z.~B. für die Arithmetik beweist, dass nicht alle Aussagen
formal bewiesen oder widerlegt werden können.
Kehren wir nun jedoch zurück zur Geometrie. Euklid hat in seiner
Kehren wir nun jedoch zurück zur Geometrie. Euklid hat in seiner
Abhandlung \enquote{Die Elemente} ein Axiomensystem für die Geometrie
aufgestellt.
aufgestellt.
\textbf{Euklids Axiome}
\begin{itemize}
@ -39,11 +39,11 @@ aufgestellt.
\item \textbf{Kreis} (um jeden Punkt mit jedem Radius)
\item Je zwei rechte Winkel sind gleich (Isometrie, Bewegung)
\item Parallelenaxiom von Euklid:\xindex{Parallelenaxiom}\\
Wird eine Gerade so von zwei Geraden geschnitten, dass die
Wird eine Gerade so von zwei Geraden geschnitten, dass die
Summe der Innenwinkel kleiner als zwei Rechte ist, dann schneiden sich
diese Geraden auf der Seite dieser Winkel.\\
\\
Man mache sich klar, dass das nur dann nicht der Fall ist,
Man mache sich klar, dass das nur dann nicht der Fall ist,
wenn beide Geraden parallel sind und senkrecht auf die erste stehen.
\end{itemize}
@ -60,7 +60,7 @@ aufgestellt.
\end{enumerate}
\item \textbf{Abstandsaxiom}\xindex{Abstandsaxiom}: Zu $P, Q, R \in X$ gibt es \label{axiom:2}
genau dann ein $g \in G$ mit $\Set{P, Q, R} \subseteq g$,
wenn gilt:
wenn gilt:
\begin{itemize}[]
\item $d(P, R) = d(P, Q) + d(Q, R)$ oder
\item $d(P, Q) = d(P, R) + d(R, Q)$ oder
@ -72,7 +72,7 @@ aufgestellt.
\begin{definition}
Sei $(X, d, G)$ eine Geometrie und seien $P, Q, R \in X$.
\begin{defenum}
\item $P, Q, R$ liegen \textbf{kollinear}\xindex{kollinear},
\item $P, Q, R$ liegen \textbf{kollinear}\xindex{kollinear},
wenn es $g \in G$ gibt mit $\Set{P, Q, R} \subseteq g$.
\item $Q$ \textbf{liegt zwischen}\xindex{liegt zwischen} $P$
und $R$, wenn $d(P, R) = d(P, Q) + d(Q, R)$
@ -103,10 +103,10 @@ aufgestellt.
\begin{beweis}\leavevmode
\begin{enumerate}[label=\alph*)]
\item \enquote{$\subseteq$} folgt direkt aus der Definition von $PR^+$ und $PR^-$\\
\enquote{$\supseteq$}: Sei $Q \in PR \Rightarrow P, Q, R$
\enquote{$\supseteq$}: Sei $Q \in PR \Rightarrow P, Q, R$
sind kollinear.\\
$\overset{\ref{axiom:2}}{\Rightarrow}
\begin{cases}
\begin{cases}
Q \text{ liegt zwischen } P \text{ und } R \Rightarrow Q \in PR\\
R \text{ liegt zwischen } P \text{ und } Q \Rightarrow Q \in PR\\
P \text{ liegt zwischen } Q \text{ und } R \Rightarrow Q \in PR
@ -133,21 +133,21 @@ aufgestellt.
\begin{enumerate}[label=§\arabic*),ref=§\arabic*,start=3]
\item \label{axiom:3}\textbf{Anordnungsaxiome}\xindex{Anordnungsaxiome}
\begin{enumerate}[label=(\roman*),ref=\theenumi{} (\roman*)]
\item \label{axiom:3.1} Zu jeder
Halbgerade $H$ mit Anfangspunkt $P \in X$ und jedem
$r \in \mdr_{\geq 0}$ gibt es genau ein
\item \label{axiom:3.1} Zu jeder
Halbgerade $H$ mit Anfangspunkt $P \in X$ und jedem
$r \in \mdr_{\geq 0}$ gibt es genau ein
$Q \in H$ mit $d(P,Q) = r$.
\item \label{axiom:3.2} Jede Gerade zerlegt
$X \setminus g = H_1 \dcup H_2$ in zwei
\item \label{axiom:3.2} Jede Gerade zerlegt
$X \setminus g = H_1 \dcup H_2$ in zwei
nichtleere Teilmengen $H_1, H_2$,
sodass für alle $A \in H_i$, $B \in H_j$ mit
$i,j \in \Set{1,2}$ gilt:
$i,j \in \Set{1,2}$ gilt:
$\overline{AB} \cap g \neq \emptyset \Leftrightarrow i \neq j$.\\
Diese Teilmengen $H_i$ heißen
\textbf{Halbebenen}\xindex{Halbebene} bzgl.
Diese Teilmengen $H_i$ heißen
\textbf{Halbebenen}\xindex{Halbebene} bzgl.
$g$.
\end{enumerate}
\item \label{axiom:4}\textbf{Bewegungsaxiom}\xindex{Bewegungsaxiom}:
\item \label{axiom:4}\textbf{Bewegungsaxiom}\xindex{Bewegungsaxiom}:
Zu $P, Q, P', Q' \in X$
mit $d(P,Q) = d(P', Q')$ gibt es mindestens 2 Isometrien $\varphi_1, \varphi_2$
mit $\varphi_i (P) = P'$ und $\varphi_i(Q) = Q'$ mit $i=1,2$.\footnote{Die \enquote{Verschiebung} von $P'Q'$ nach $PQ$ und die Isometrie, die zusätzlich an der Gerade durch $P$ und $Q$ spiegelt.}
@ -163,14 +163,14 @@ aufgestellt.
%%%%%%%%%%%%%%%%%%%%%%%%%%%%%%%%%%%%%%%%%%%%%%%%%%%%%%%%%%%%%%%%%%%%%
\begin{satz}[Satz von Pasch]\label{satz:pasch} %In Vorlesung: Bemerkung 14.5
Seien $P$, $Q$, $R$ nicht kollinear, $g \in G$ mit $g \cap \Set{P, Q, R} = \emptyset$
und $g \cap \overline{PQ} \neq \emptyset$.
und $g \cap \overline{PQ} \neq \emptyset$.
Dann ist entweder $g \cap \overline{PR} \neq \emptyset$ oder
Dann ist entweder $g \cap \overline{PR} \neq \emptyset$ oder
$g \cap \overline{QR} \neq \emptyset$.
\end{satz}
Dieser Satz besagt, dass Geraden, die eine Seite eines Dreiecks
(also nicht nur eine Ecke) schneiden, auch eine weitere Seite
Dieser Satz besagt, dass Geraden, die eine Seite eines Dreiecks
(also nicht nur eine Ecke) schneiden, auch eine weitere Seite
schneiden.
\begin{beweis}
@ -182,7 +182,7 @@ schneiden.
\end{beweis}
\begin{bemerkung}\label{kor:beh3}
Sei $P, Q \in X$ mit $P \neq Q$ sowie $A, B \in X \setminus PQ$
Sei $P, Q \in X$ mit $P \neq Q$ sowie $A, B \in X \setminus PQ$
mit $A \neq B$.
Außerdem seien $A$ und $B$ in der selben Halbebene bzgl. $PQ$ sowie
$Q$ und $B$ in der selben Halbebene bzgl. $PA$.
@ -197,8 +197,8 @@ schneiden.
\label{fig:geometry-5}
\end{figure}
Auch \cref{kor:beh3} lässt sich umgangssprachlich sehr viel
einfacher ausdrücken: Die Diagonalen eines konvexen Vierecks
Auch \cref{kor:beh3} lässt sich umgangssprachlich sehr viel
einfacher ausdrücken: Die Diagonalen eines konvexen Vierecks
schneiden sich.
\begin{beweis}%In Vorlesung: Behauptung 3
@ -217,7 +217,7 @@ schneiden sich.
$\overline{AP'}$ liegt in der anderen Halbebene
bzgl. $PA \Rightarrow C \notin \overline{P'A} \Rightarrow C \in \overline{AQ}$
\end{enumerate}
Da $C \in PB^+$ und $C \in \overline{AQ}$ folgt nun direkt:
Da $C \in PB^+$ und $C \in \overline{AQ}$ folgt nun direkt:
$\emptyset \neq \Set{C} \subseteq PB^+ \cap \overline{AQ} \qed$
\end{beweis}
@ -286,7 +286,7 @@ schneiden sich.
\begin{bemerkung}\label{kor:beh2'}
Sei $(X, d, G)$ eine Geometrie, die \ref{axiom:1}~-~\ref{axiom:3}
erfüllt, $P, Q \in X$ mit $P \neq Q$ und $\varphi$ eine Isometrie mit
erfüllt, $P, Q \in X$ mit $P \neq Q$ und $\varphi$ eine Isometrie mit
$\varphi(P) = P$ und $\varphi(Q) = Q$.
Dann gilt $\varphi(S) = S\;\;\;\forall S \in PQ$.
@ -302,7 +302,7 @@ schneiden sich.
&\overset{\mathclap{\ref{axiom:3.1}}}{\Rightarrow} \varphi(S) = S
\end{align*}
$\qed$
$\qed$
\end{beweis}
\begin{proposition}\label{satz:14.4}%In Vorlesung: Satz 14.4
@ -310,7 +310,7 @@ schneiden sich.
gibt es zu $P, P', Q, Q'$ mit $d(P, Q) = d(P', Q')$ höchstens
zwei Isometrien mit $\varphi(P) = P'$ und $\varphi(Q) = Q'$
Aus den Axiomen folgt, dass es in
Aus den Axiomen folgt, dass es in
der Situation von \ref{axiom:4} höchstens zwei Isometrien mit
$\varphi_i(P) = P'$ und $\varphi_i(Q) = Q'$ gibt.
\end{proposition}
@ -332,11 +332,11 @@ schneiden sich.
Nun zu den Beweisen der Teilaussagen:
\begin{enumerate}[label=(Teil \roman*),ref=(Teil \roman*)]
\item Sei $R \in X \setminus PQ$. Von den drei Punkten
\item Sei $R \in X \setminus PQ$. Von den drei Punkten
$\varphi_1(R), \varphi_2(R), \varphi_3(R)$ liegen zwei
in der selben Halbebene bzgl. $P'Q' = \varphi_i(PQ)$.
\Obda seien $\varphi_1(R)$ und $\varphi_2(R)$ in der
\Obda seien $\varphi_1(R)$ und $\varphi_2(R)$ in der
selben Halbebene.
Es gilt: $\begin{aligned}[t]
@ -392,7 +392,7 @@ schneiden sich.
Also gilt insbesondere $\varphi(\triangle A'B'C') = \triangle ABC$. $\qed$
\end{beweis}
\begin{bemerkung}[WSW-Kongruenzsatz]\xindex{Kongruenzsatz!WSW}%
Sei $(X, d, G)$ eine Geometrie, die \ref{axiom:1}~-~\ref{axiom:4} erfüllt.
Seien außerdem $\triangle ABC$ und $\triangle A'B'C'$ Dreiecke, für die gilt:
@ -424,15 +424,15 @@ schneiden sich.
\begin{definition}\label{def:14.8}%In Vorlesung: 14.8
\begin{defenum}
\item \label{def:14.8a} Ein \textbf{Winkel}\xindex{Winkel} ist ein Punkt $P \in X$
\item \label{def:14.8a} Ein \textbf{Winkel}\xindex{Winkel} ist ein Punkt $P \in X$
zusammen mit $2$ Halbgeraden mit Anfangspunkt $P$.\\
Man schreibt: $\angle R_1 P R_2$ bzw. $\angle R_2 P R_1$\footnote{Für dieses Skript gilt: $\angle R_1 P R_2 = \angle R_2 P R_1$. Also sind insbesondere alle Winkel $ \leq 180^\circ$.}
\item Zwei Winkel sind \textbf{gleich}, wenn es eine Isometrie gibt,
\item Zwei Winkel sind \textbf{gleich}, wenn es eine Isometrie gibt,
die den einen Winkel auf den anderen abbildet.
\item \label{def:14.8c} $\angle R_1' P' R_2'$ heißt \textbf{kleiner} als
$\angle R_1 P R_2$, wenn es eine Isometrie $\varphi$
gibt, mit $\varphi(P') = P$, $\varphi(P'R'^{+}_{1}) = PR_{1}^{+}$
und $\varphi(R_2')$ liegt in der gleichen Halbebene
und $\varphi(R_2')$ liegt in der gleichen Halbebene
bzgl. $PR_1$ wie $R_2$ und in der gleichen Halbebene
bzgl. $PR_2$ wie $R_1$
\item \label{def:14.8d} Im Dreieck $\triangle PQR$ gibt es \textbf{Innenwinkel}\xindex{Innenwinkel} und
@ -455,7 +455,7 @@ schneiden sich.
\end{figure}
\begin{bemerkung}\label{bem:14.9}%In Vorlesung: Bemerkung 14.9
In einem Dreieck ist jeder Innenwinkel kleiner als jeder nicht
In einem Dreieck ist jeder Innenwinkel kleiner als jeder nicht
anliegende Außenwinkel.
\end{bemerkung}
@ -481,7 +481,7 @@ schneiden sich.
\caption{Situation aus \cref{bem:14.9}}
\end{figure}
Es gilt: $d(Q,M) = d(M,R)$ und $d(P,M) = d(M,A)$ sowie
Es gilt: $d(Q,M) = d(M,R)$ und $d(P,M) = d(M,A)$ sowie
$\angle PMR = \angle AMQ \Rightarrow \triangle MRQ$ ist
kongruent zu $\triangle AMQ$, denn eine der beiden Isometrien, die
$\angle PMR$ auf $\angle AMQ$ abbildet, bildet $R$ auf $Q$ und
@ -496,7 +496,7 @@ schneiden sich.
\begin{proposition}[Existenz der Parallelen]\label{prop:14.7}%In Vorlesung: Proposition 14.7
Sei $(X, d, G)$ eine Geometrie mit den Axiomen \ref{axiom:1}~-~\ref{axiom:4}.
Dann gibt es zu jeder Geraden $g \in G$ und jedem Punkt $P \in X \setminus g$
Dann gibt es zu jeder Geraden $g \in G$ und jedem Punkt $P \in X \setminus g$
mindestens eine Parallele $h \in G$ mit $P \in h$ und $g \cap h = \emptyset$.
\end{proposition}
@ -535,7 +535,7 @@ Halbebene bzgl. $PQ$ liegt wie $R$.
\begin{figure}[htp]
\centering
\includegraphics[width=0.4\linewidth, keepaspectratio]{figures/Spherical_triangle_3d_opti.png}
\includegraphics[width=0.4\linewidth, keepaspectratio]{figures/Spherical_triangle_3d_opti.png}
\caption{In der sphärischen Geometrie gibt es, im Gegensatz zur euklidischen Geometrie, Dreiecke mit drei $90^\circ$-Winkeln.}
\label{fig:spherical-triangle}
\end{figure}
@ -567,7 +567,7 @@ Sei im Folgenden \enquote{$\IWS$} die \enquote{Innenwinkelsumme}.
Sei $\alpha$ ein Innenwinkel von $\triangle$.
\begin{behauptung}
Es gibt ein Dreieck $\triangle'$ mit
Es gibt ein Dreieck $\triangle'$ mit
$\IWS(\triangle') = \IWS(\triangle)$ und einem Innenwinkel
$\alpha' \leq \frac{\alpha}{2}$.
@ -579,8 +579,8 @@ Sei im Folgenden \enquote{$\IWS$} die \enquote{Innenwinkelsumme}.
\end{behauptung}
\begin{beweis}
Es seien $A, B, C \in X$ und $\triangle $ das Dreieck mit den
Eckpunkten $A, B, C$ und $\alpha$ sei der Innenwinkel bei $A$,
Es seien $A, B, C \in X$ und $\triangle $ das Dreieck mit den
Eckpunkten $A, B, C$ und $\alpha$ sei der Innenwinkel bei $A$,
$\beta$ der Innenwinkel bei $B$ und $\gamma$ der Innenwinkel bei $C$.
Sei $M$ der Mittelpunkt der Strecke $\overline{BC}$. Sei außerdem
@ -613,7 +613,7 @@ Sei im Folgenden \enquote{$\IWS$} die \enquote{Innenwinkelsumme}.
\end{figure}
\begin{beweis}
Sei $g$ eine Parallele von $AB$ durch $C$.
Sei $g$ eine Parallele von $AB$ durch $C$.
\begin{itemize}
\item Es gilt $\alpha' = \alpha$ wegen \cref{prop:14.7}.
@ -639,7 +639,7 @@ WSW.\xindex{Kongruenzsatz!SWW}
\label{fig:hyperbolische-geometrie-2}
\end{figure}
Der Beweis wird hier nicht geführt. Für Beweisvorschläge wäre ich
Der Beweis wird hier nicht geführt. Für Beweisvorschläge wäre ich
dankbar.
\begin{figure}[htp]
@ -730,10 +730,10 @@ $\xRightarrow{\text{Strahlensatz}} \frac{a}{h_c} = \frac{c}{h_a} \rightarrow a \
\item $(\mdr^2, d_\text{Euklid})$ ist offensichtlich eine euklidische Ebene.
\item Sei $(X,d)$ eine euklidische Ebene und $g_1, g_2$ Geraden
in $X$, die sich in einem Punkt $0$ im rechten Winkel
schneiden.
schneiden.
Sei $P \in X \setminus (g_1 \cup g_2)$ ein Punkt und $P_X$ der
Fußpunkt des Lots von $P$ auf $g_1$ (vgl. \cref{ub11:aufg3.c})
Sei $P \in X \setminus (g_1 \cup g_2)$ ein Punkt und $P_X$ der
Fußpunkt des Lots von $P$ auf $g_1$ (vgl. \cref{ub11:aufg3.c})
und $P_Y$ der Fußpunkt des Lots von $P$ auf $g_2$.
Sei $x_P := d(P_X, 0)$ und $y_P := d(P_Y, 0)$.
@ -754,11 +754,11 @@ $\xRightarrow{\text{Strahlensatz}} \frac{a}{h_c} = \frac{c}{h_a} \rightarrow a \
\label{fig:14.13.0.1}
\end{figure}
Sei $h:X \rightarrow \mdr^2$ eine Abbildung mit
$h(P) := (x_P, y_P)$
Dadurch wird $h$ auf dem Quadranten
definiert, in dem $P$ liegt, d.~h.
definiert, in dem $P$ liegt, d.~h.
\[\forall Q \in X \text{ mit } \overline{PQ} \cap g_1 = \emptyset = \overline{PQ} \cap g_2\]
Fortsetzung auf ganz $X$ durch konsistente Vorzeichenwahl.
@ -833,7 +833,7 @@ $\xRightarrow{\text{Strahlensatz}} \frac{a}{h_c} = \frac{c}{h_a} \rightarrow a \
Betrachte nun $z_1$ und $z_2$ als Punkte in der
euklidischen Ebene. Die Mittelsenkrechte zu diesen
Punkten schneidet die $x$-Achse. Alle Punkte auf
der Mittelsenkrechten zu $z_1$ und $z_2$ sind gleich
der Mittelsenkrechten zu $z_1$ und $z_2$ sind gleich
weit von $z_1$ und $z_2$ entfernt. Daher ist
der Schnittpunkt mit der $x$-Achse der Mittelpunkt
eines Kreises durch $z_1$ und $z_2$ (vgl. \cref{fig:hyperbolische-geometrie-axiom-1-2})
@ -867,7 +867,7 @@ $\xRightarrow{\text{Strahlensatz}} \frac{a}{h_c} = \frac{c}{h_a} \rightarrow a \
\underline{Zu zeigen:}
$\forall A \in H_i$, $B \in H_j$ mit
$i,j \in \Set{1,2}$ gilt:
$i,j \in \Set{1,2}$ gilt:
$\overline{AB} \cap g \neq \emptyset \Leftrightarrow i \neq j$\\
\enquote{$\Leftarrow$}: $A \in H_1, B \in H_2: \overline{AB} \cap g \neq \emptyset$
@ -882,10 +882,10 @@ $\xRightarrow{\text{Strahlensatz}} \frac{a}{h_c} = \frac{c}{h_a} \rightarrow a \
Sei $h$ die Gerade, die durch $A$ und $B$ geht.
Da $A,B \notin g$, aber $A, B \in h$ gilt, haben $g$ und $h$
Da $A,B \notin g$, aber $A, B \in h$ gilt, haben $g$ und $h$
insbesondere
mindestens einen unterschiedlichen Punkt. Aus \ref{axiom:1.1} folgt, dass sich
$g$ und $h$ in höchstens einen Punkt schneiden. Sei $C$ dieser
$g$ und $h$ in höchstens einen Punkt schneiden. Sei $C$ dieser
Punkt.
Aus $A,B \notin g$ folgt: $C \neq A$ und $C \neq B$. Also liegt
@ -903,8 +903,8 @@ $\xRightarrow{\text{Strahlensatz}} \frac{a}{h_c} = \frac{c}{h_a} \rightarrow a \
\end{beweis}
\begin{definition}\xindex{Möbiustransformation}%
Es seien $a,b,c,d \in \mdr$ mit $ad - bc \neq 0$ und
$\sigma: \mdc \rightarrow \mdc$ eine Abbildung definiert durch
Es seien $a,b,c,d \in \mdr$ mit $ad - bc \neq 0$ und
$\sigma: \mdc \rightarrow \mdc$ eine Abbildung definiert durch
\[\sigma(z) := \frac{az + b}{cz+d}\]
$\sigma$ heißt \textbf{Möbiustransformation}.
@ -953,7 +953,7 @@ $\xRightarrow{\text{Strahlensatz}} \frac{a}{h_c} = \frac{c}{h_a} \rightarrow a \
&= \frac{a(a'z+b')+b(c'z+d')}{c(a'z+b') + d(c'z+d')}\\
&= \frac{(aa'+bc')z + ab' + bd'}{(ca'+db')z + cb' + dd'}\\
&= \begin{pmatrix}aa'+bc'&ab'+bd'\\ca'+db'&cb'+dd'\end{pmatrix} \circ z\\
&= \left ( \begin{pmatrix}a&b\\c&d\end{pmatrix} \cdot \begin{pmatrix}a'&b'\\c'&d'\end{pmatrix} \right ) \circ z
&= \left ( \begin{pmatrix}a&b\\c&d\end{pmatrix} \cdot \begin{pmatrix}a'&b'\\c'&d'\end{pmatrix} \right ) \circ z
\end{align*}
\item Es gilt $\sigma(z) = (-\sigma)(z)$ für alle $\sigma \in \SL_2(\mdr)$
und $z \in \mdh$.
@ -975,7 +975,7 @@ $\xRightarrow{\text{Strahlensatz}} \frac{a}{h_c} = \frac{c}{h_a} \rightarrow a \
Daher genügt es zu zeigen, dass man mit $A_{\lambda}$, $B_t$ und $C$ alle Matrizen
aus $\SL_2(\mdr)$ erzeugen kann, genügt es also von einer beliebigen
Matrix durch Multiplikation mit Matrizen der Form $A_{\lambda}$,
Matrix durch Multiplikation mit Matrizen der Form $A_{\lambda}$,
$B_t$ und $C$ die Einheitsmatrix zu generieren.
Sei also
@ -1056,9 +1056,9 @@ $\xRightarrow{\text{Strahlensatz}} \frac{a}{h_c} = \frac{c}{h_a} \rightarrow a \
\begin{definition}\xindex{Doppelverhältnis}%In Vorlesung: Def+Prop 15.4
Seien $z_1, z_2, z_3, z_4 \in \mdc$ paarweise verschieden.
Dann heißt
Dann heißt
\[\DV(z_1, z_2, z_3, z_4) := \frac{\frac{z_1 - z_4}{z_1 - z_2}}{\frac{z_3 - z_4}{z_3 - z_2}} = \frac{(z_1 - z_4) \cdot (z_3 - z_2)}{(z_1 - z_2) \cdot (z_3 - z_4)}\]
\textbf{Doppelverhältnis} von
\textbf{Doppelverhältnis} von
$z_1, \dots, z_4$.
\end{definition}
@ -1071,7 +1071,7 @@ $\xRightarrow{\text{Strahlensatz}} \frac{a}{h_c} = \frac{c}{h_a} \rightarrow a \
oder wenn zwei der $z_i$ gleich sind.
\item $\DV(0, 1, \infty, z_4) = z_4$ (Der Fall $z_4 \in \Set{0, 1, \infty}$ ist zugelassen).
\item \label{bem:15.4d} Für $\sigma \in \PSL_2(\mdc)$ und $z_1, \dots, z_4 \in \mdc \cup \Set{\infty}$
ist
ist
\[\DV(\sigma(z_1), \sigma(z_2), \sigma(z_3), \sigma(z_4)) = \DV(z_1, z_2, z_3, z_4)\]
und für $\sigma(z) = \frac{1}{\overline{z}}$ gilt
\[\DV(\sigma(z_1), \sigma(z_2), \sigma(z_3), \sigma(z_4)) = \overline{\DV(z_1, z_2, z_3, z_4)}\]
@ -1110,7 +1110,7 @@ $\xRightarrow{\text{Strahlensatz}} \frac{a}{h_c} = \frac{c}{h_a} \rightarrow a \
Durch Einsetzen ergibt sich $\DV(z_1, \dots, z_4)=1$.
\end{itemize}
Im Fall, dass ein $z_i = \infty$ ist, ist
Im Fall, dass ein $z_i = \infty$ ist, ist
entweder $\DV(0, 1, \infty, z_4) = 0$ oder $\DV(0, 1, \infty, z_4) \pm \infty$
\item $\DV(0, 1, \infty, z_4) = \frac{(0- z_4) \cdot (\infty - 1)}{(0 -1) \cdot (\infty - z_4)} = \frac{z_4 \cdot (\infty - 1)}{\infty - z_4} = z_4$
\item Wenn jemand diesen Beweis führt, bitte an info@martin-thoma.de schicken.%TODO

View file

@ -12,16 +12,16 @@
\section{Krümmung von Kurven}\label{sec:Kurvenkrümmung}
\begin{definition}%In Vorlesung: Def.+Bem. 16.1
Sei $\gamma: I = [a, b] \rightarrow \mdr^n$ eine Kurve.
\begin{defenum}
\item Die Kurve $\gamma$ heißt
\item Die Kurve $\gamma$ heißt
\textbf{durch Bogenlänge parametrisiert}\xindex{parametrisiert!durch Bogenlänge},
wenn gilt:
\[\|\gamma'(t)\|_2 = 1 \;\;\; \forall t \in I\]
Dabei ist $\gamma'(t) = \left (\gamma_1'(t), \gamma_2'(t), \dots, \gamma_n'(t) \right)$.
\item $l(\gamma) = \int_a^b \|\gamma'(t)\| \mathrm{d} t$ heißt
\textbf{Länge von $\gamma$}\xindex{Kurve!Länge einer}.
\end{defenum}
\end{defenum}
\end{definition}
\begin{bemerkung}[Eigenschaften von Kurven I]%In Vorlesung: Def.+Bem. 16.1
@ -29,7 +29,7 @@
\begin{bemenum}
\item Ist $\gamma$ durch Bogenlänge parametrisiert, so ist $l(\gamma) = b-a$.
\item \label{bem:16.1d} Ist $\gamma$ durch Bogenlänge parametrisiert, so ist
\item \label{bem:16.1d} Ist $\gamma$ durch Bogenlänge parametrisiert, so ist
$\gamma'(t)$ orthogonal zu $\gamma''(t)$ für alle $t \in I$.
\end{bemenum}
\end{bemerkung}
@ -85,8 +85,8 @@ Da $n(t)$ und $\gamma''(t)$ nach \cref{bem:16.1d} linear
da gilt:
\begin{align*}
\langle n(t), \gamma'(t) \rangle &=
\left \langle
\langle n(t), \gamma'(t) \rangle &=
\left \langle
\begin{pmatrix}- \cos \frac{t}{r}\\ - \sin \frac{t}{r}\end{pmatrix},
\begin{pmatrix}- \sin \frac{t}{r}\\ \cos \frac{t}{r}\end{pmatrix}
\right \rangle\\
@ -128,7 +128,7 @@ Da $n(t)$ und $\gamma''(t)$ nach \cref{bem:16.1d} linear
Also gilt:
\[\det(\gamma'(t), n(t), b(t)) = 1\]
$b(t)$ heißt \textbf{Binormalenvektor}\xindex{Binormalenvektor},
die Orthonormalbasis
die Orthonormalbasis
\[\Set{\gamma'(t), n(t), b(t)}\]
heißt \textbf{begleitendes Dreibein}\xindex{Dreibein!begleitendes}.
\end{defenum}
@ -185,16 +185,16 @@ für eine $C^\infty$-Funktion $f: \mdr^3 \rightarrow \mdr$.
\begin{beweis}\leavevmode
\begin{enumerate}[label=\alph*)]
\item \label{bew:tangentialebene.a} $J_F$ ist eine $3 \times 2$-Matrix, die mit einem $2 \times 1$-Vektor
multipliziert wird. Das ist eine lineare Abbildung und aus der
multipliziert wird. Das ist eine lineare Abbildung und aus der
linearen Algebra ist bekannt, das das Bild ein Vektorraum ist.
Da $\rang(J_F) = 2$, ist auch $\dim (T_s S) = 2$.
\item Hier kann man wie in \cref{bew:tangentialebene.a} argumentieren
\item $T_s S = \{x \in \mdr^3 | \exists \text{parametrisierte Kurve }
\gamma:[- \varepsilon, + \varepsilon] \rightarrow S
\text{ für ein } \varepsilon > 0
\item $T_s S = \{x \in \mdr^3 | \exists \text{parametrisierte Kurve }
\gamma:[- \varepsilon, + \varepsilon] \rightarrow S
\text{ für ein } \varepsilon > 0
\text{ mit } \gamma(0) = s \text{ und } \gamma'(0) = x
\}$\\
Wenn jemand diesen Beweis führt, bitte an info@martin-thoma.de
Wenn jemand diesen Beweis führt, bitte an info@martin-thoma.de
schicken.%TODO
\item Sei $x \in T_s S, \gamma:[-\varepsilon, +\varepsilon] \rightarrow S$
eine parametrisierte Kurve mit $\varepsilon > 0$ und $\gamma'(0) = s$,
@ -231,7 +231,7 @@ Im Folgenden werden diese Begriffe jedoch synonym benutzt.
von $s$ und eine lokale Parametrisierung $F: U \rightarrow V$
von $S$ um $s$, sodass auf $F(U) = V \cap S$
ein stetiges Normalenfeld existiert.
\item $S$ ist genau dann orientierbar, wenn es einen
\item $S$ ist genau dann orientierbar, wenn es einen
differenzierbaren Atlas von $S$ aus lokalen Parametrisierungen
$F_i: U_i \rightarrow V_i,\;i \in I$ gibt, sodass
für alle $i, j \in F$ und alle $s \in V_i \cap V_j \cap S$
@ -256,7 +256,7 @@ Im Folgenden werden diese Begriffe jedoch synonym benutzt.
\begin{figure}[htp]\xindex{Möbiusband}
\centering
\includegraphics[width=0.5\linewidth, keepaspectratio]{figures/moebius-strip.pdf}
\includegraphics[width=0.5\linewidth, keepaspectratio]{figures/moebius-strip.pdf}
\caption{Möbiusband}
\label{fig:moebius-strip}
\end{figure}
@ -266,7 +266,7 @@ Im Folgenden werden diese Begriffe jedoch synonym benutzt.
Sei $S$ eine reguläre Fläche, $s \in S$, $n(s)$ ist ein Normalenvektor
in $s$, $x \in T_s S$, $\|x\| = 1$.
Sei $E$ der von $x$ und $n(s)$ aufgespannte 2-dimensionale
Sei $E$ der von $x$ und $n(s)$ aufgespannte 2-dimensionale
Untervektorraum von $\mdr^3$.
Dann gibt es eine Umgebung $V \subseteq \mdr^3$ von $s$, sodass
@ -277,7 +277,7 @@ Im Folgenden werden diese Begriffe jedoch synonym benutzt.
\end{bemerkung}
\begin{beweis}
\enquote{Satz über implizite Funktionen}\footnote{Siehe z.~B.
\enquote{Satz über implizite Funktionen}\footnote{Siehe z.~B.
\url{https://github.com/MartinThoma/LaTeX-examples/tree/master/documents/Analysis\%20II}}
\end{beweis}
@ -362,18 +362,18 @@ Im Folgenden werden diese Begriffe jedoch synonym benutzt.
\end{beweis}
\begin{bemerkung}%In Vorlesung: Bem.+Def. 18.6
Sei $S$ eine reguläre Fläche und $n=n(s)$ ein Normalenvektor an
Sei $S$ eine reguläre Fläche und $n=n(s)$ ein Normalenvektor an
$S$ in $s$.
Sei $T_{s}^{1} S = \Set{x \in T_s S | \|x\| = 1} \cong S^1$.
Dann ist
Dann ist
\[ \kappanor^n(s): T^1_s S \rightarrow \mdr, \;\;\; x \mapsto \kappanor(s,x)\]
eine glatte Funktion und
eine glatte Funktion und
$\Bild \kappanor^n(s)$ ist ein abgeschlossenes Intervall.
\end{bemerkung}
\begin{definition}\xindex{Hauptkrümmung}\xindex{Gauß-Krümmung}%In Vorlesung: Bem.+Def. 18.6
Sei $S$ eine reguläre Fläche und $n=n(s)$ ein Normalenvektor an
Sei $S$ eine reguläre Fläche und $n=n(s)$ ein Normalenvektor an
$S$ in $s$.
\begin{defenum}
@ -388,7 +388,7 @@ Im Folgenden werden diese Begriffe jedoch synonym benutzt.
\end{definition}
\begin{bemerkung}%In Vorlesung: Bem.+Def. 18.6
Ersetzt man $n$ durch $-n$, so gilt:
Ersetzt man $n$ durch $-n$, so gilt:
\begin{align*}
\kappanor^{-n}(s, x) &= - \kappanor^n(x)\; \forall x \in T_s^1 S\\
@ -457,7 +457,7 @@ $s$. Weiter sei $p := F^{-1}(s)$.
\item \label{bem:19.1a} Die Einschränkung des Standardskalarproduktes des $\mdr^3$ auf
$T_s S$ macht $T_s S$ zu einem euklidischen Vektorraum.
\item $\Set{D_p F(e_1), D_p F(e_2)}$ ist eine Basis von $T_s S$.
\item Bzgl. der Basis $\Set{D_p F(e_1), D_p F(e_2)}$ hat das
\item Bzgl. der Basis $\Set{D_p F(e_1), D_p F(e_2)}$ hat das
Standardskalarprodukt aus \cref{bem:19.1a} die Darstellungsmatrix
$I_S$.
\item $g_{i,j}(s)$ ist eine differenzierbare Funktion von $s$.
@ -465,7 +465,7 @@ $s$. Weiter sei $p := F^{-1}(s)$.
\end{bemerkung}
\begin{bemerkung}
\[\det(I_S) = \left \| \frac{\partial F}{\partial u_1}(p) \times \frac{\partial F}{\partial u_2}(p) \right \|^2\]
\[\det(I_S) = \left \| \frac{\partial F}{\partial u_1}(p) \times \frac{\partial F}{\partial u_2}(p) \right \|^2\]
\end{bemerkung}
\begin{beweis}\leavevmode
@ -495,7 +495,7 @@ $s$. Weiter sei $p := F^{-1}(s)$.
\begin{defenum}
\item Das Differential $\mathrm{d} A = \sqrt{\det (I)} \mathrm{d} u_1 \mathrm{d} u_2$
heißt \textbf{Flächenelement} von $S$ bzgl. der Parametrisierung $F$.
\item \label{def:berechenbares-integral}Für eine Funktion $f: V \rightarrow \mdr$ heißt
\item \label{def:berechenbares-integral}Für eine Funktion $f: V \rightarrow \mdr$ heißt
\[\int_V f \mathrm{d} A := \int_U f(\underbrace{F(u_1, u_2)}_{=: s}) \sqrt{\det I(s)} \mathrm{d} u_1 \mathrm{d} u_2\]
der \textbf{Wert des Integrals} von $f$ über $V$, falls das Integral rechts
existiert.
@ -535,7 +535,7 @@ $s$. Weiter sei $p := F^{-1}(s)$.
\begin{propenum}
\item \label{prop:5.1a} $n$ induziert für jedes $s \in S$ eine lineare Abbildung $d_s n: T_s S \rightarrow T_{n(s)} S^2$
durch
durch
\[d_s n(x) = \frac{\mathrm{d}}{\mathrm{d} t} n (\underbrace{s \text{\enquote{+}} tx}_{\mathclap{\text{Soll auf Fläche $S$ bleiben}}}) \Bigr |_{t=0}\]
Die Abbildung $d_s n$ heißt \textbf{Weingarten-Abbildung}
\item $T_{n(s)} S^2 = T_s S$.
@ -560,7 +560,7 @@ $s$. Weiter sei $p := F^{-1}(s)$.
Sei $x_i = D_p F(e_i) = \frac{\partial F}{\partial u_i} (p)\;\;\; i = 1,2$
\underline{Beh.:}
\underline{Beh.:}
$\langle x_i, d_s n(x_j) \rangle = \langle \frac{\partial^2 F}{\partial u_i \partial u_j} (p), d_s n (x_i) \rangle$
$\Rightarrow \langle \frac{\partial^2 F}{\partial u_i \partial u_j} (p), d_s n (x_i) \rangle = \langle x_j, d_s n (x_i) \rangle$
@ -597,9 +597,9 @@ $s$. Weiter sei $p := F^{-1}(s)$.
\begin{beweis}
Nach \cref{def:18.4} ist $\kappanor(s, \gamma) = \langle \gamma''(0), n(s) \rangle$.
Nach Voraussetzung gilt
Nach Voraussetzung gilt
\[n(\gamma(t)) \perp \gamma'(t) \Leftrightarrow \langle \gamma''(0), n(s) \rangle = 0\]
Die Ableitung nach $t$ ergibt
Die Ableitung nach $t$ ergibt
\begin{align*}
0 &= \frac{\mathrm{d}}{\mathrm{d}t}(\langle n (\gamma(t)), \gamma'(t))\\
&= \left \langle \frac{\mathrm{d}}{\mathrm{d}t} n(\gamma(t)) \Bigr |_{t=0}, \gamma'(0) \right \rangle + \langle n(s), \gamma''(0) \rangle\\
@ -629,7 +629,7 @@ $s$. Weiter sei $p := F^{-1}(s)$.
Eigenvektoren $y_1, y_2$ von $II_s$. Ist $x \in T_s S$, $\|x\| = 1$,
so gibt es $\varphi \in [0,2\pi)$ mit $x = \cos \varphi \cdot y_1 + \sin \varphi \cdot y_2$.
Seien $\lambda_1, \lambda_2$ die Eigenwerte von $II_s$, also
Seien $\lambda_1, \lambda_2$ die Eigenwerte von $II_s$, also
$II_s(y_i, y_i) = \lambda_i$. Dann gilt:
\begin{align*}
II_s (x,x) &= \cos^2 \varphi \lambda_1 + \sin^2 \varphi \lambda_2\\

View file

@ -20,13 +20,13 @@
$(X, \fT_X)$ ist also nicht hausdorffsch.
\textbf{Teilaufgabe c)} Nach Bemerkung \ref{Trennungseigenschaft}
sind metrische Räume hausdorffsch. Da $(X, \fT_X)$ nach (b) nicht
sind metrische Räume hausdorffsch. Da $(X, \fT_X)$ nach (b) nicht
hausdorffsch ist, liefert die Kontraposition der Trennungseigenschaft,
dass $(X, \fT_X)$ kein metrischer Raum sein kann.
\end{solution}
\begin{solution}[\ref{ub1:aufg4}]
\textbf{Teilaufgabe a)}
\textbf{Teilaufgabe a)}
\textbf{Beh.:} $\forall a \in \mdz: \Set{a}$ ist abgeschlossen.
@ -35,7 +35,7 @@
Wenn jemand diese Aufgabe gemacht hat, bitte die Lösung an info@martin-thoma.de
schicken.%TODO
\textbf{Teilaufgabe b)}
\textbf{Teilaufgabe b)}
\textbf{Beh.:} $\Set{-1, 1}$ ist nicht offen
@ -50,7 +50,7 @@
in dieser Topologie offen sein $\Rightarrow \Set{-1,1}$ ist
nicht offen. $\qed$
\textbf{Teilaufgabe c)}
\textbf{Teilaufgabe c)}
\textbf{Beh.:} Es gibt unendlich viele Primzahlen.
@ -58,7 +58,7 @@
Annahme: Es gibt nur endlich viele Primzahlen $p \in \mdp$
Dann ist
Dann ist
\[\mdz \setminus \Set{-1, +1} \overset{\text{FS d. Arithmetik}}= \bigcup_{p \in \mdp} U_{0,p}\]
endlich. Das ist ein Widerspruch zu $|\mdz|$ ist unendlich und
$|\Set{-1,1}|$ ist endlich. $\qed$
@ -67,7 +67,7 @@
\begin{solution}[\ref{ub2:aufg4}]
\begin{enumerate}[label=(\alph*)]
\item \textbf{Beh.:} Die offenen Mengen von $P$ sind
Vereinigungen von Mengen der Form
Vereinigungen von Mengen der Form
\[\prod_{j \in J} U_j \times \prod_{i \in \mdn, i \neq j} P_i\]
wobei $J \subseteq \mdn$ endlich und $U_j \subseteq P_j$
offen ist.
@ -79,7 +79,7 @@
$\forall{j \in J}$
eine Basis der Topologie.
Damit sind die offenen
Damit sind die offenen
Mengen von $P$ Vereinigungen von Mengen der obigen
Form. $\qed$
\end{beweis}
@ -95,10 +95,10 @@
für alle $i \in \mdn$ gilt entweder $p_i(Z) \subseteq \Set{0}$
oder $p_i(Z) \subseteq \Set{1}$. Es sei $z_i \in \Set{0,1}$
so, dass $p_i(Z) \subseteq \Set{z_i}$ für alle $i \in \mdn$.
Dann gilt also:
Dann gilt also:
\[\underbrace{p_i(x)}_{= x_i} = z_i = \underbrace{p_i(y)}_{= y_i} \forall i \in \mdn\]
Somit folgt: $x = y \qed$
\end{beweis}
\end{enumerate}
\end{solution}
@ -107,9 +107,9 @@
\begin{enumerate}[label=(\alph*)]
\item \textbf{Beh.:} $\GL_n(\mdr)$ ist nicht kompakt.\\
\textbf{Bew.:} $\det: \GL_n(\mdr) \rightarrow \mdr \setminus \Set{0}$
ist stetig. Außerdem ist
$\det(\GL_n(\mdr)) = \mdr \setminus \Set{0}$ nicht
kompakt. $\overset{\ref{kor:5.6}}{\Rightarrow}$
ist stetig. Außerdem ist
$\det(\GL_n(\mdr)) = \mdr \setminus \Set{0}$ nicht
kompakt. $\overset{\ref{kor:5.6}}{\Rightarrow}$
$\GL_n(\mdr)$ ist nicht kompakt. $\qed$
\item \textbf{Beh.:} $\SL_1(\mdr)$ ist nicht kompakt, für $n > 1$ ist $\SL_n(\mdr)$ kompakt.\\
\textbf{Bew.:} Für $\SL_1(\mdr)$ gilt:
@ -119,7 +119,7 @@
$\SL_n(\mdr) \subseteq \GL_n(\mdr)$ lässt sich mit einer
Teilmenge des $\mdr^{n^2}$ identifizieren. Nach \cref{satz:heine-borel}
sind diese genau dann kompakt, wenn sie beschränkt und
sind diese genau dann kompakt, wenn sie beschränkt und
abgeschlossen sind. Definiere nun für für $n \in \mdn_{\geq 2}, m \in \mdn$:
\[A_m = \text{diag}_n(m, \frac{1}{m}, \dots, 1)\]
Dann gilt: $\det A_m = 1$, d.~h. $A_m \in \SL_n(\mdr)$,
@ -138,7 +138,7 @@
nachgelesen werden.
\begin{definition}\xindex{Homomorphismus}%
Seien $(G, *)$ und $(H, \circ)$ Gruppen und
Seien $(G, *)$ und $(H, \circ)$ Gruppen und
$\varphi:G \rightarrow H$ eine Abbildung.
$\varphi$ heißt \textbf{Homomorphismus}, wenn
@ -161,16 +161,16 @@
\begin{solution}[\ref{ub3:meinsExtra2}]
Die Definition einer Isotopie kann auf \cpageref{def:Isotopie} nachgelesen
werden, die einer Isometrie auf \cpageref{def:Isometrie}.
\begin{definition}\xindex{Isomorphismus}%
Seien $(G, *)$ und $(H, \circ)$ Gruppen und
Seien $(G, *)$ und $(H, \circ)$ Gruppen und
$\varphi:G \rightarrow H$ eine Abbildung.
$\varphi$ heißt \textbf{Isomorphismus}, wenn $\varphi$ ein bijektiver
Homomorphismus ist.
\end{definition}
Eine Isotopie ist also für Knoten definiert, Isometrien machen nur in
Eine Isotopie ist also für Knoten definiert, Isometrien machen nur in
metrischen Räumen Sinn und ein Isomorphismus benötigt eine Gruppenstruktur.
\end{solution}
@ -180,7 +180,7 @@
\textbf{Beh.:} $M$ ist wegzusammehängend $\gdw M$ ist zusammenhängend
\begin{beweis}
\enquote{$\Rightarrow$}: Da $M$ insbesondere ein
topologischer Raum ist folgt diese Richtung direkt
topologischer Raum ist folgt diese Richtung direkt
aus \cref{kor:wegzusammehang-impliziert-zusammenhang}.
\enquote{$\Leftarrow$}: Seien $x,y \in M$ und
@ -192,7 +192,7 @@
\item $Z^C := \Set{\tilde{z} \in M | \nexists \text{Weg von } x \text{ nach } \tilde{z}}$ ist offen
Da $M$ eine Mannigfaltigkeit ist, existiert zu jedem
$\tilde{z} \in Z^C$ eine offene und wegzusammenhängende Umgebung
$\tilde{z} \in Z^C$ eine offene und wegzusammenhängende Umgebung
$U_{\tilde{z}} \subseteq M$.
Es gilt sogar $U_{\tilde{z}} \subseteq Z^C$, denn
@ -226,7 +226,7 @@
Da $(\mdr \setminus \Set{0}) \cup \Set{0_1}$ homöomorph
zu $\mdr$ ist, exisitert ein Weg $\gamma_1$ von $0_1$
zu einem beliebigen Punkt $a \in \mdr \setminus \Set{0}$.
Da $(\mdr \setminus \Set{0}) \cup \Set{0_2}$ ebenfalls
homöomorph zu $\mdr$ ist, existiert außerdem ein Weg
$\gamma_2$ von $a$ nach $0_2$. Damit existiert ein
@ -241,18 +241,18 @@
% \textbf{Beh.:} $H_k = \begin{cases}\mdr &\text{für } k\in \Set{0,1}\\
% 0 &\text{für } k \geq 2$
% \newcommand{\triangleSimplizialkomplex}{\mathord{\includegraphics[height=5ex]{figures/triangleSimplizialkomplex.pdf}}}
% \textbf{Bew.:} $S^1$ ist homöomorph zum Simplizialkomplex
% \textbf{Bew.:} $S^1$ ist homöomorph zum Simplizialkomplex
% $X = \triangleSimplizialkomplex$, d.~h. dem Rand
% von $\Delta^2$. Es gilt:
% \[X = \Set{\underbrace{v_0, v_1, v_2}_{A_0(X)}, \underbrace{\Delta (v_1, v_2)}_{=: a_0}, \underbrace{\underbrace{\Delta (v_0, v_2)}_{=: a_1}, \underbrace{\Delta(v_0, v_1)}_{=: a_2}}_{A_1(X)}}\]
% Damit folgt:
% Damit folgt:
% \begin{enumerate}
% \item Für $k \geq 2$ ist $C_k(X) \cong 0$, da es in diesen
% \item Für $k \geq 2$ ist $C_k(X) \cong 0$, da es in diesen
% Dimensionen keine Simplizes gibt, d.~h. $A_k(X) = \emptyset$ gilt.\\
% Also: $H_k(X) \cong 0 \; \forall k \geq 2$
% \item $C_0(X) = \Set{\sum_{i=0}^2 c_i v_i | c_i \in \mdr}$, da
% \item $C_0(X) = \Set{\sum_{i=0}^2 c_i v_i | c_i \in \mdr}$, da
% $A_0(x)$ Basis von $C_0(X)$ ist;\\
% $C_1(X) = \Set{\sum_{i=0}^2 c_i a_i | c_i \in \mdr}$, da
% $C_1(X) = \Set{\sum_{i=0}^2 c_i a_i | c_i \in \mdr}$, da
% $A_1(X)$ Basis von $C_1(X)$ ist.
% \item Für die Randabbildungen $d_i: C_i(X) \rightarrow C_{i-1}(X)$ gilt:
% $d_0 \equiv 0$, $d_1: C_1(X) \rightarrow C_0(X)$ ist definiert durch
@ -287,7 +287,7 @@
$\angle BCA \xRightarrow{\crefabbr{bem:14.9}} \measuredangle BC' A > \measuredangle BCA$\\
$\Rightarrow \measuredangle BCA < \measuredangle BC' A = \measuredangle ABC' < \measuredangle ABC $
Sei umgekehrt $\measuredangle ABC > \measuredangle BCA$,
kann wegen 1. Teil von \cref{ub11:aufg3.b} nicht
kann wegen 1. Teil von \cref{ub11:aufg3.b} nicht
$d(A,B) > d(A,C)$ gelten.\\
Wegen \cref{ub11:aufg3.a} kann nicht $d(A,B) = d(A,C)$
gelten.\\
@ -343,7 +343,7 @@
d(B, C) &= d(B', C')
\end{align*}
Sei $\varphi$ die Isometrie mit $\varphi(A) = A'$, $\varphi(B) = B'$ und
Sei $\varphi$ die Isometrie mit $\varphi(A) = A'$, $\varphi(B) = B'$ und
$\varphi(C')$ liegt in der selben Halbebene bzgl. $AB$ wie $C$. Diese
Isometrie existiert wegen \ref{axiom:4}.

View file

@ -5,23 +5,23 @@ wird, die die Inhalte noch lernen, sind sehr wahrscheinlich einige
Fehler im Skript. Das können Übertragungsfehler, Tippfehler oder
Verständnisprobleme sein.
Verbesserungsvorschläge (auch wenn es nur einzelne Textsetzungsprobleme oder
Verbesserungsvorschläge (auch wenn es nur einzelne Textsetzungsprobleme oder
Rechtschreibfehler sind) bitte immer direkt melden oder verbessern!
Den Verbesserungsvorschlag kann man
* entweder direkt selbst umsetzen und einen pull request machen oder
* mir per Email (info@martin-thoma.de) schicken.
Den Verbesserungsvorschlag kann man
* entweder direkt selbst umsetzen und einen pull request machen oder
* mir per Email (info@martin-thoma.de) schicken.
Ich werde dann versuchen die Verbesserungsvorschläge zeitnah einzuarbeiten.
Zeichnungen
===========
Das erstellen der Zeichnungen ist sehr zeitaufwendig. Das ist der
Das erstellen der Zeichnungen ist sehr zeitaufwendig. Das ist der
Grund, warum manchmal nur ein "TODO" im Dokument steht.
Ihr könnt mir gerne Zeichnungen schicken (entweder schön auf Papier
Zeichnen und abfotographieren / einscannen oder schon mit Inscape /
Gimp / ... oder sogar mit TikZ erstellen).
Gimp / ... oder sogar mit TikZ erstellen).
Akzeptable Formate sind: .jpg, .pdf, .svg, .png, .gif, .tex, .sketch
Alles andere kann ich vermutlich nicht einbinden.
@ -37,8 +37,8 @@ Rechtliches
===========
Die Autoren kann man über Git ermitteln. Ich schreibe meist nur den
Tafelanschrieb der Vorlesung ab; eventuell noch mit ein paar
Notizen meinerseits. Wenn mir Verbesserungsvorschläge per Email
geschickt werden, ist der Autor sowie das Datum der Email in der
Notizen meinerseits. Wenn mir Verbesserungsvorschläge per Email
geschickt werden, ist der Autor sowie das Datum der Email in der
Commit-Nachricht von Git zu sehen.
Bilder habe ich entweder selbst erstellt oder von tex.stackexchange.com.

View file

@ -6,19 +6,19 @@ der Vorlesung von Prof.~Dr.~Herrlich sowie die Mitschriften einiger
Übungen und Tutorien.
Das Skript ist kostenlos über \href{http://martin-thoma.com/geotopo/}{martin-thoma.com/geotopo}
verfügbar. Wer es gerne in A5 (Schwarz-Weiß, Ringbindung) für 10 Euro hätte,
verfügbar. Wer es gerne in A5 (Schwarz-Weiß, Ringbindung) für 10 Euro hätte,
kann mir eine E-Mail schicken (info@martin-thoma.de).
\section*{Danksagungen}
An dieser Stelle möchte ich Herrn~Prof.~Dr.~Herrlich für einige
Korrekturvorschläge und einen gut strukturierten Tafelanschrieb
An dieser Stelle möchte ich Herrn~Prof.~Dr.~Herrlich für einige
Korrekturvorschläge und einen gut strukturierten Tafelanschrieb
danken, der als Vorlage für dieses Skript diente. Tatsächlich basiert
die Struktur dieses Skripts auf der Vorlesung von Herrn~Prof.~Dr.~Herrlich
und ganze Abschnitte konnten direkt mit \LaTeX{} umgesetzt werden.
Vielen Dank für die Erlaubnis, Ihre Inhalte in diesem Skript einbauen
zu dürfen!
Vielen Dank auch an Frau Lenz und Frau Randecker, die es mir erlaubt
Vielen Dank auch an Frau Lenz und Frau Randecker, die es mir erlaubt
haben, ihre Übungsaufgaben und Lösungen zu benutzen.
Jérôme Urhausen hat durch viele Verbesserungsvorschläge und Beweise zu einer erheblichen
@ -70,11 +70,11 @@ der Umgang mit komplexen Zahlen $\mdc$, deren Betrag, Folgen und
Häufungspunkten nicht weiter schwer fallen.
Diese Vorkenntnisse werden vor allem in \enquote{Analysis I} vermittelt.
Außerdem wird vorausgesetzt, dass (affine) Vektorräume, Faktorräume,
Außerdem wird vorausgesetzt, dass (affine) Vektorräume, Faktorräume,
lineare Unabhängigkeit, der Spektralsatz und der projektive Raum $\praum(\mdr)$ aus
\enquote{Lineare Algebra I} bekannt sind. In \enquote{Lineare Algebra II}
wird der Begriff der Orthonormalbasis eingeführt.
Obwohl es nicht vorausgesetzt wird, könnte es von Vorteil sein
\enquote{Einführung in die Algebra und Zahlentheorie} gehört zu
\enquote{Einführung in die Algebra und Zahlentheorie} gehört zu
haben.

View file

@ -46,10 +46,10 @@
%%
\usepackage{../shortcuts}
\hypersetup{
pdfauthor = {Martin Thoma},
pdfkeywords = {Geometrie und Topologie},
pdftitle = {Geometrie und Topologie - Definitionen}
\hypersetup{
pdfauthor = {Martin Thoma},
pdfkeywords = {Geometrie und Topologie},
pdftitle = {Geometrie und Topologie - Definitionen}
}
\allowdisplaybreaks

View file

@ -38,10 +38,10 @@
\usepackage[left=10mm,right=10mm, top=2mm, bottom=10mm]{geometry}
\usepackage{../shortcuts}
\hypersetup{
pdfauthor = {Martin Thoma},
pdfkeywords = {Geometrie und Topologie},
pdftitle = {Geometrie und Topologie - Definitionen}
\hypersetup{
pdfauthor = {Martin Thoma},
pdfkeywords = {Geometrie und Topologie},
pdftitle = {Geometrie und Topologie - Definitionen}
}
\allowdisplaybreaks

View file

@ -18,15 +18,15 @@
tick align=outside,
%minor tick num=-3,
enlargelimits=true]
\addplot[domain=0:12, red, thick,samples=500] {1/3*x^1.5};
\addplot[domain=0:12, dotted, orange, thick,samples=500] {1*x^1.5};
\addplot[domain=0:12, dashed, blue, thick,samples=500] {2*x^1.5};
\addplot[domain=0:12, red, thick,samples=500] {1/3*x^1.5};
\addplot[domain=0:12, dotted, orange, thick,samples=500] {1*x^1.5};
\addplot[domain=0:12, dashed, blue, thick,samples=500] {2*x^1.5};
\addplot[domain=0:12, red, thick,samples=500] {-1/3*x^1.5};
\addplot[domain=0:12, dotted, orange, thick,samples=500] {-1*x^1.5};
\addplot[domain=0:12, dashed, blue, thick,samples=500] {-2*x^1.5};
\addplot[domain=0:12, red, thick,samples=500] {-1/3*x^1.5};
\addplot[domain=0:12, dotted, orange, thick,samples=500] {-1*x^1.5};
\addplot[domain=0:12, dashed, blue, thick,samples=500] {-2*x^1.5};
\addlegendentry{$a=\frac{1}{3}$}
\addlegendentry{$a=1$}
\addlegendentry{$a=2$}
\end{axis}
\end{axis}
\end{tikzpicture}

View file

@ -27,5 +27,5 @@
}
]
\addplot3[surf] {y*y-x*x*x};
\end{axis}
\end{axis}
\end{tikzpicture}

View file

@ -14,13 +14,13 @@
\tkzDefLine[orthogonal=through P,/tikz/overlay](O,X) \tkzGetPoint{helper}
\tkzInterLL(O,X)(P,helper) \tkzGetPoint{xp}
\draw [decorate,decoration={brace,amplitude=4pt,mirror}]
(O) -- (xp) node [black,midway,xshift=0cm, yshift=-0.3cm]
(O) -- (xp) node [black,midway,xshift=0cm, yshift=-0.3cm]
{\footnotesize $x_P$};
\tkzDefLine[orthogonal=through P,/tikz/overlay](O,Y) \tkzGetPoint{helper}
\tkzInterLL(O,Y)(P,helper) \tkzGetPoint{yp}
\draw [decorate,decoration={brace,amplitude=4pt}]
(O) -- (yp) node [black,midway,xshift=-0.4cm]
(O) -- (yp) node [black,midway,xshift=-0.4cm]
{\footnotesize $y_P$};
\tkzDrawPolygon(O,xp,P,yp)

View file

@ -10,13 +10,13 @@
\tkzDefLine[orthogonal=through P,/tikz/overlay](O,X) \tkzGetPoint{helper}
\tkzInterLL(O,X)(P,helper) \tkzGetPoint{xp}
\draw [decorate,decoration={brace,amplitude=4pt,mirror}]
(O) -- (xp) node [black,midway,xshift=0cm, yshift=-0.3cm]
(O) -- (xp) node [black,midway,xshift=0cm, yshift=-0.3cm]
{\footnotesize $x_P$};
\tkzDefLine[orthogonal=through P,/tikz/overlay](O,Y) \tkzGetPoint{helper}
\tkzInterLL(O,Y)(P,helper) \tkzGetPoint{yp}
\draw [decorate,decoration={brace,amplitude=4pt}]
(O) -- (yp) node [black,midway,xshift=-0.4cm]
(O) -- (yp) node [black,midway,xshift=-0.4cm]
{\footnotesize $y_P$};
\tkzDrawPolygon(O,xp,P,yp)

View file

@ -18,9 +18,9 @@
\path[name path=trpath] (tl) -- (fr);
\path[name path=tlpath] (tr) -- (fl);
\draw[name intersections={of=brpath and rbpath}] (intersection-1)coordinate (br){};
\draw[name intersections={of=blpath and lbpath}] (intersection-1)coordinate (bl){};
\draw[name intersections={of=trpath and tlpath}] (intersection-1)coordinate (tb){};
\draw[name intersections={of=brpath and rbpath}] (intersection-1)coordinate (br){};
\draw[name intersections={of=blpath and lbpath}] (intersection-1)coordinate (bl){};
\draw[name intersections={of=trpath and tlpath}] (intersection-1)coordinate (tb){};
\shade[right color=gray!10, left color=black!50, shading angle=105] (tf) -- (bf) -- (bl) -- (tl) -- cycle;
\shade[left color=gray!10, right color=black!50, shading angle=75] (tf) -- (bf) -- (br) -- (tr) -- cycle;

View file

@ -63,5 +63,5 @@
\node at (axis cs:0,3) [anchor=east] {$y$};
\node at (axis cs:2,0) [anchor=north] {$x$};
\end{axis}
\end{axis}
\end{tikzpicture}

View file

@ -43,5 +43,5 @@
\node[blue] at (axis cs:0,3) [anchor=east] {$x_2$};
\node[blue] at (axis cs:2,0) [anchor=north] {$x_1$};
\end{axis}
\end{axis}
\end{tikzpicture}

View file

@ -9,7 +9,7 @@
},
}
\begin{tikzpicture}
\draw[->] (-1.5,0) -- (5.5,0) node [below] {$\mathbb{R}$};
\foreach \x in {-1,...,5}

View file

@ -23,5 +23,5 @@
% Draw axis text
\node at (axis cs:-1,0.5) [anchor=east] {$\mathfrak{B}_r(0) = $};
\end{axis}
\end{axis}
\end{tikzpicture}

View file

@ -12,10 +12,10 @@
\draw[grid] (\x,-0.5) -- (\x,2.5);
\draw[grid] (-0.5,\y) -- (2.5,\y);
}
%draw the axes
\draw[axis] (-1,0,0) -- (3,0,0) node[anchor=west]{$y$};
\draw[axis] (0,-1,0) -- (0,3,0) node[anchor=west]{$x$};
\end{tikzpicture}

View file

@ -1,7 +1,7 @@
\begin{tikzpicture}[scale=.5, z={(.707,.3)}]
\draw (2,3,2) -- (0,0,0) -- (4,0,0) -- (4,0,4) -- (2,3,2)
\draw (2,3,2) -- (0,0,0) -- (4,0,0) -- (4,0,4) -- (2,3,2)
-- (4,0,0);
\draw[color=gray, style=dashed] (2,3,2) -- (0,0,4)
\draw[color=gray, style=dashed] (2,3,2) -- (0,0,4)
-- (0,0,0);
\draw[color=gray, style=dashed] (0,0,4) -- (4,0,4);
\end{tikzpicture}

View file

@ -28,14 +28,14 @@
enlargelimits=true,
tension=0.08]
% plot the stirling-formulae
\addplot[domain=-4:8, red, thick,samples=500] {0.5*x};
\addplot[domain=-2:2, red, thick,samples=500] {2*x};
\addplot[domain=-4:4, red, thick,samples=500] {x};
\addplot[domain=-4:8, red, thick,samples=500] {-0.5*x};
\addplot[domain=-4:8, red, thick,samples=500] {0.5*x};
\addplot[domain=-2:2, red, thick,samples=500] {2*x};
\addplot[domain=-4:4, red, thick,samples=500] {x};
\addplot[domain=-4:8, red, thick,samples=500] {-0.5*x};
\addplot[color=red,only marks,mark=o]
plot coordinates {
(1.5,3)
(1.5,1.5)
};
\end{axis}
\end{axis}
\end{tikzpicture}

View file

@ -6,7 +6,7 @@
\node (Rright) at (6,0) {};
\draw[dashed,very thick] (Pleft) -- (P);
\draw[dotted,very thick] (P) -- (R) -- (Rright);
\draw [thick,decoration={brace,mirror,raise=0.2cm},decorate] (Pleft) -- (P) node [pos=0.5,anchor=north,yshift=-0.25cm] {$PR^-$};
\draw [thick,decoration={brace,mirror,raise=0.2cm},decorate] (P) -- (R) node [pos=0.5,anchor=north,yshift=-0.25cm] {$\overline{PR}$};
\draw [thick,decoration={brace,mirror,raise=0.8cm},decorate] (P) -- (Rright) node [pos=0.5,anchor=north,yshift=-0.85cm] {$PR^+$};
\draw [thick,decoration={brace,mirror,raise=0.2cm},decorate] (Pleft) -- (P) node [pos=0.5,anchor=north,yshift=-0.25cm] {$PR^-$};
\draw [thick,decoration={brace,mirror,raise=0.2cm},decorate] (P) -- (R) node [pos=0.5,anchor=north,yshift=-0.25cm] {$\overline{PR}$};
\draw [thick,decoration={brace,mirror,raise=0.8cm},decorate] (P) -- (Rright) node [pos=0.5,anchor=north,yshift=-0.85cm] {$PR^+$};
\end{tikzpicture}

View file

@ -29,5 +29,5 @@
\node at (axis cs:0.8,1.2) [anchor=-90] {$y$};
\draw (axis cs:0.8,1.2) circle[radius=0.6];
\addplot[mark=*] coordinates {(0.8,1.2)};
\end{axis}
\end{axis}
\end{tikzpicture}

View file

@ -9,7 +9,7 @@
},
}
\begin{tikzpicture}
\draw[->] (-0.5,0) -- (1.5,0) node [below] {$\mathbb{R}$};
\foreach \x in {0,...,1}

View file

@ -63,7 +63,7 @@
\addplot[hatchcolor=red,mark=none, pattern=custom north west lines, draw=none] coordinates {(4.5, 0) (4.5,5) (5.5,5) (5.5,0) };
\addplot[red,mark=none, thick] coordinates {(4.5, 0) (4.5,5)};
\addplot[red,mark=none, thick] coordinates {(5.5, 0) (5.5,5)};
\addplot[mark=none, dashed] coordinates {(1, 0) (1,3)};
\addplot[mark=none, dashed] coordinates {(5, 0) (5,3)};
@ -78,5 +78,5 @@
\node[red] at (axis cs:1,-0.3) [anchor=north] {$U_1 \times X_2$};
\node[red] at (axis cs:5,-0.3) [anchor=north] {$U_2 \times X_2$};
\end{axis}
\end{axis}
\end{tikzpicture}

View file

@ -23,5 +23,5 @@
(0,0) (-1,1) (-2,2) (-1,3) (0, 3) (1, 4)};
\addplot[mark=none, blue, smooth cycle, thick, tension=3] coordinates {%
(0,0) (-1,1) (-2,2) (-1,3) (0, 3) (1, 4)};
\end{axis}
\end{axis}
\end{tikzpicture}

View file

@ -21,5 +21,5 @@
\addplot[domain=0:2.5, red, thick,samples=20] {-x+2.5};
\node[point,label={[label distance=0cm]45:$e_0$}] at (axis cs:2.5,0) {};
\node[point,label={[label distance=0cm]0:$e_1$}] at (axis cs:0,2.5) {};
\end{axis}
\end{axis}
\end{tikzpicture}

View file

@ -20,5 +20,5 @@
\node (b)[point,label={[label distance=0cm]5:$e_1$}] at (axis cs:0,2.5) {};
\node (c)[point,label={[label distance=0cm]0:$e_2$}] at (axis cs:2,2) {};
\draw[thick,fill=orange!50] (a.center) -- (b.center) -- (c.center) -- cycle;
\end{axis}
\end{axis}
\end{tikzpicture}

View file

@ -21,6 +21,6 @@
\addplot[domain=0.0105:0.011, blue, thick,samples=20] {10};
\addlegendentry{$\{(x, \sin(\frac{1}{x})) \in X \times Y\}$}
\addlegendentry{$(-1,1) \subseteq Y$}
\end{axis}
\end{axis}
\draw[ultra thick,blue] (0,0.5) -- (0,4);
\end{tikzpicture}

View file

@ -9,7 +9,7 @@
\node (e)[point] at (0,2) {};
\node (f)[point] at (4,2) {};
\end{scope}
\node (p)[point,label={[label distance=0cm]5:$P$}] at (1.5,0.5) {};
\draw[pattern=north east lines] (a.center) -- (b.center) -- (c.center) -- cycle;

View file

@ -9,7 +9,7 @@
\node (e)[point] at (0,2) {};
\node (f)[point] at (4,2) {};
\end{scope}
\node (p)[point,label={[label distance=0cm]5:$P$}] at (1.5,0.5) {};
\draw[pattern=north east lines] (a.center) -- (p.center) -- (b.center) -- cycle;

View file

@ -14,7 +14,7 @@
\begin{scope}[decoration={
markings,
mark=at position 0.6 with {\arrow{>}}}
]
]
\draw[postaction={decorate}] (a.center) -- (b.center);
\draw[postaction={decorate}] (d.center) -- (c.center);
\end{scope}
@ -22,7 +22,7 @@
\begin{scope}[decoration={
markings,
mark=at position 0.55 with {\arrow{>>}}}
]
]
\draw[postaction={decorate}] (b.center) -- (c.center);
\draw[postaction={decorate}] (a.center) -- (d.center);
\end{scope}

View file

@ -21,7 +21,7 @@
\begin{scope}[decoration={
markings,
mark=at position 0.6 with {\arrow{>}}}
]
]
\draw[postaction={decorate}] (a.center) -- (b.center);
\draw[postaction={decorate}] (d.center) -- (c.center);
\end{scope}
@ -29,7 +29,7 @@
\begin{scope}[decoration={
markings,
mark=at position 0.55 with {\arrow{>>}}}
]
]
\draw[postaction={decorate}] (b.center) -- (c.center);
\draw[postaction={decorate}] (a.center) -- (d.center);
\end{scope}

View file

@ -38,7 +38,7 @@
\begin{scope}[decoration={
markings,
mark=at position 0.6 with {\arrow{>}}}
]
]
\draw[postaction={decorate}] (a.center) -- (b.center);
\draw[postaction={decorate}] (d.center) -- (c.center);
\end{scope}
@ -46,7 +46,7 @@
\begin{scope}[decoration={
markings,
mark=at position 0.55 with {\arrow{>>}}}
]
]
\draw[postaction={decorate}] (b.center) -- (c.center);
\draw[postaction={decorate}] (a.center) -- (d.center);
\end{scope}

View file

@ -36,7 +36,7 @@
\begin{scope}[decoration={
markings,
mark=at position 0.6 with {\arrow{>}}}
]
]
\draw[postaction={decorate}] (a.center) -- (b.center);
\draw[postaction={decorate}] (d.center) -- (c.center);
\end{scope}
@ -44,7 +44,7 @@
\begin{scope}[decoration={
markings,
mark=at position 0.55 with {\arrow{>>}}}
]
]
\draw[postaction={decorate}] (b.center) -- (c.center);
\draw[postaction={decorate}] (a.center) -- (d.center);
\end{scope}

View file

@ -13,8 +13,8 @@
\node at ($(C')+(0,0.4)$) {$C'$};
\node at ($(B)+(0.2,-0.2)$) {$B$};
\node at ($(C)+(0.28,0.5)$) {$C$};
\tkzDrawPolygon[ultra thick,color=blue,fill=blue!20](A,B',C')
\tkzDrawPolygon[line width=0.3pt,color=red,fill=red!20](A,B,C)
\tkzDrawPolygon[ultra thick,color=blue,fill=blue!20](A,B',C')
\tkzDrawPolygon[line width=0.3pt,color=red,fill=red!20](A,B,C)
\tkzDrawPoints(A,B',C',B,C)
\tkzLabelSegment[below,red](A,B){$c$}
\tkzLabelSegment[left,red](A,C){$b$}

View file

@ -18,7 +18,7 @@
\tkzDrawLines(A,B)
\tkzDrawLine[dashed,color=orange,add=0.5 and 0.2](F,P)
\tkzDrawLine[dashed,color=blue,add=0.5 and 0.2](G,P)
%
%
\tkzLabelPoint[below left](A){$A$}
\tkzLabelPoint[below left](G){$G$}
\tkzLabelPoint[above left](P){$P$}

View file

@ -18,8 +18,8 @@
enlargelimits=true,
tension=0.08]
% plot the stirling-formulae
\addplot[domain=-4:8, red, thick,samples=500] {0.5*x};
\addplot[domain=-2:2, red, thick,samples=500] {2*x};
\addplot[domain=-4:8, red, thick,samples=500] {-0.5*x};
\end{axis}
\addplot[domain=-4:8, red, thick,samples=500] {0.5*x};
\addplot[domain=-2:2, red, thick,samples=500] {2*x};
\addplot[domain=-4:8, red, thick,samples=500] {-0.5*x};
\end{axis}
\end{tikzpicture}

View file

@ -29,5 +29,5 @@
\addplot[mark=none] coordinates {(\i,-0.2) (\i,5.2)};
}
\addplot[mark=none] coordinates {(0,2) (5,2)};
\end{axis}
\end{axis}
\end{tikzpicture}

View file

@ -68,7 +68,7 @@ in dem Erstellen dieses Skripts steckt:
|07.02.2014 | 11:15 - 11:20 | 5 | Definitionen vereinfacht
|07.02.2014 | 11:35 - 11:45 | 10 | Definition "operiert durch Homöomorphismen" korrigiert
|07.02.2014 | 15:00 - 15:30 | 30 | Verbesserungsvorschläge von Jérôme Urhausen, Email vom 08.02.2014, umgesetzt.
|07.02.2014 | 15:30 - 15:45 | 15 | Verbesserungen
|07.02.2014 | 15:30 - 15:45 | 15 | Verbesserungen
|07.02.2014 | 19:30 - 21:20 | 110 | Textsetzung, kleine Fehler und Verbesserung eines Bildes
|10.02.2014 | 10:30 - 11:05 | 35 | Formulierung in Definitionen vereinfacht; Textsetzung
|10.02.2014 | 11:05 - 11:20 | 15 | Verbesserungsvorschläge von Marco, Email 1 vom 10.02.2014, umgesetzt.
@ -92,6 +92,6 @@ in dem Erstellen dieses Skripts steckt:
|20.02.2014 | 12:00 - 13:00 | 60 | Verbesserungsvorschläge von Jonathan (Facebook, 20.02.2014) eingearbeitet.
|20.02.2014 | 13:00 - 13:45 | 45 | Verbesserungsvorschläge von Jérôme Urhausen, Email 1 vom 20.02.2014, umgesetzt.
|20.02.2014 | 19:30 - 20:15 | 45 | Verbesserungsvorschläge von Jérôme Urhausen, Email 2 vom 20.02.2014, umgesetzt.
| Zwischenstand | --- | --- | 6081 Minuten => Über 100 Stunden!
| Zwischenstand | --- | --- | 6081 Minuten => Über 100 Stunden!
|17.03.2014 | 16:00 - 18:00 | 120 | Textsetzung
|19.03.2014 | 08:00 - 10:00 | 120 | Verbesserung des Symbolverzeichnisses

View file

@ -1,4 +1,4 @@
Zu diesem Skript haben einige Leute beigetragen. Die Personen, die am
Zu diesem Skript haben einige Leute beigetragen. Die Personen, die am
meisten beigetragen haben, stehen direkt im Skript unter "Danksagungen".
Hier ist eine (hoffentlich bald) vollständige Liste der Mitwirkenden (alphabetisch geordnet) mit

View file

@ -4,7 +4,7 @@ bitte ich um eine Email.
Konventionen
============
* `\mathbb{N}` sollte vermieden werden. Stattdessen wird
* `\mathbb{N}` sollte vermieden werden. Stattdessen wird
`\mathbb{N}_0` und `\mathbb{N}_+` verwendet.
* `\subset` sollte vermieden werden. Stattdessen wird
`\subseteq` bzw. `\subsetneq` verwendet.